Sunteți pe pagina 1din 26

the foreign law as a question of fact, and it is then necessary to

MARRIAGE, FAMILY LAW AND ADOPTION prove the alleged foreign marriage by convincing evidence. A
Philippine marriage followed by 23 years of uninterrupted marital
1. Adong v. Cheong Seng Gee, 43 Phil. 43 (1922) life, should not be impugned and discredited, after the death of the
GR No.18081 March 3, 1922 husband through an alleged prior Chinese marriage, “save upon
proof so clear, strong and unequivocal as to produce a moral
FACTS: conviction of the existence of such impediment.” A marriage alleged
Cheong Boo, a native of China died in Zamboanga, Philippine Islands to have been contracted in China and proven mainly by a so-called
matrimonial letter held not to be valid in the Philippines
on August 5, 1919 and left property worth nearly P100,000 which is
now being claimed by two parties –
(1) Cheong Seng Gee who alleged that he was a legitimate child by
marriage contracted by Cheong Boo with Tan Bit in China in 1985, 2. Ramirez v. Gmur, 42 Phil. 855 (1918)
and
FACTS: ​Samuel Bischoff Werthmuller, a native of Switzerland, but for
(2) Mora Adong who alleged that she had been lawfully married to
Cheong Boo in 1896 in Basilan, Philippine Islands and had two many years a resident of Philippines, died testate in Iloilo on June
daughters with the deceased namely Payang and Rosalia. The 29, 1913. A few days after his death, the will was offered for probate
conflicting claims to Cheong Boo’s estate were ventilated in the in CFI Iloilo and was subsequently admitted.
lower court that ruled that Cheong Seng Gee failed to sufficiently
establish the Chinese marriage through a mere letter testifying that 1. His widow, Dona Ana Ramirez, was named executrix. According
Cheong Boo and Tan Bit married each other but that because to the will, the decedent’s estate was bequeathed to his widow,
Cheong Seng Gee had been admitted to the Philippine Islands as the except for a property in Thun, Switzerland, which was devised to the
son of the deceased, he should share in the estate as a natural child. decedent’s brothers and sisters
With reference to the allegations of Mora Adong and her daughters,
the trial court reached the conclusion that the marriage between 2. It appears that while the decedent had no children with
Adong and Cheong Boo had been adequately proven but that under Ramirez, he ignored possible claims of two sets of children, both to
the laws of the Philippine Islands it could not be held to be a lawful his natural daughter, Leona Castro
marriage and thus the daughter Payang and Rosalia would inherit as
natural children. The lower court believes that Mohammedan 3. Based on the baptismal entry in Bacolod, Leona Castro was
marriages are not valid under the Philippine Island’s laws this as an born on April 11, 1875, her mother Felisa Castro and father
Imam as a solemnizing officer and under Quaranic laws.
“unknown.” On the margin of this record, there is additional
ISSUES: ​Whether or not the Chinese marriage between Cheong Boo annotation (escritura or public document) that she was recognized
and Tan Dit is valid? by Samuel Bischoff on June 22, 1877. Bischoff and his family raised
Whether or not the Mohammedan marriage between Cheong Boo Leona and he treated her as his own.
and Mora Adong is valid?
4. Leona married Frederick von Kauffman in 1895 in Hong Kong.
HELD:
From this marriage, 3 children were born: Elena, Federico and
The Supreme Court found the (1) Chinese marriage not proved and Ernesto
Chinaman Cheong Seng Gee has only the rights of a natural child
while 5. Subsequently, Leona went with Frederick to Thun, Switzerland,
(2) it found the Mohammedan marriage to be proved and to be to improve her health and live there for a few years. Later she
valid, thus giving to the widow Mora Adong and the legitimate informed her husband that she wanted a separation. As such,
children Payang and Rosalia the rights accruing to them under the Kauffman went to Paris, France to obtain a divorce from his wife
law. The Supreme Court held that marriage in this jurisdiction is not under French laws. A divorce decree was then issued on Jan 5, 1905
only a civil contract but it is a new relation, an instruction in the in favor of Kauffman against Leona, in default. Though the record
maintenance of which the public is deeply interested. The
presumption as to marriage is that every shows that Leona was residing in Paris, there is no evidence that she
had acquired permanent domicile in that city
intendment of the law leans toward legalizing matrimony. Persons
dwelling together in apparent matrimony are presumed, in the 6. Leona later cohabited with her physician, Dr. Ernest Emil Mory
absence of counter-presumption or evidence special to the case, to (in charge of the sanatorium in Switzerland) in London. They later
be in fact married. The reason is that such is the common order of married in London in 1905. It appears that Mory had previously
society, and if the parties were not what they thus hold themselves
out as being, they would be living in the constant violation of been married to one Helena Wolpman but later divorced. Prior to
decency of the law. As to retroactive force, marriage laws is in the Mory’s marriage to Leona, they had a daughter Leontina Elizabeth.
nature of a curative provision intended to safeguard society by In 106, they had Carmen Maria and in 1909, Esther. Leona died on
legalizing prior marriages. Public policy should aid acts intended to Oct 6, 1910
validate marriages and should retard acts intended to invalidate
marriages. This as for public policy, the courts can properly incline 7. Respondent Otto Gmur appeared as guardian of the 3 Mory
the scales of their decision in favor of that solution which will most claimants, while Frederick von Kauffman appeared as the guardian
effectively promote the public policy. That is the true construction of his 3 children
which will best carry legislative intention into effect. Sec. IV of the
Marriage law provides that “all marriages contracted outside the 8. Both sets of children anchor their claim on the contention that
islands, which would be valid by the laws of the country in which the Leona was the recognized natural daughter of Samuel Bischoff, as a
same were contracted, are valid in these islands. To establish a valid forced heir, the claimants are entitled to 1/3 interest in Bischoff’s
foreign marriage pursuant to this comity provision, it is first
necessary to prove before the courts of the Islands the existence of estate
9. On the other hand, Ramirez contended that Leona Castro had would have been his forced heir, he having died after the Civil Code
never been recognized at all by the decedent during his lifetime took effect; and as such forced heir, she would have been entitled to
1/3 of the inheritance.
10. On the part of the Mory claimants, they argue that the marriage
between Leona and Kauffman was dissolved by the decree of ISSUE: WON the Kauffman claimants are entitled to participate in
divorce granted by the Paris court in Jan 5, 1905 and that the the division of the estate
subsequent marriage between Leona and Mory was in all respects HELD: Yes. Since the Kauffman children were born during the
valid and the children legitimate offspring of Leona marriage of Leona and Kauffman, it follows that they are entitled to
11. On the part of the Kauffman claimants, they insisted that the participate in the inheritance which would have devolved upon their
decree of divorce was invalid and they alone are the legitimate mother, if she had survived the testator.
offspring of Leona who are entitled to participate in the division of ISSUE: WON the divorce decree granted by the Paris court can be
the estate of Bischoff recognized in the Philippines
ISSUE: WON Leona is the recognized natural daughter of Samuel HELD: No. The divorce decree in question cannot be recognized as
Bischoff valid in the Philippines. The French tribunal has no jurisdiction to
entertain an action for the dissolution of a marriage contracted in
the Philippines by a person domiciled her; such marriage being
HELD: Yes. It is proved that prior to her marriage with Kauffman, indissoluble under the laws then prevailing in this country.
Leona was in an uninterrupted enjoyment of the de facto status of a
natural child and was treated as such by Bischoff and his kindred.
The proof of tacit recognition is full and complete. The evidence shows that both Kauffman and Leona are domiciled in
Iloilo, Philippines and that their departure to Switzerland was for
medical purpose, and that Kauffman went to Paris in 1904 to obtain
From the memorandum made by Padre Ferrero in the record of a divorce without an intention to establish permanent residence in
birth as well as in his testimony, it appears that Bischoff executed a that city.
notarized document recognizing Leona as his daughter. While the
note itself was not presented in evidence, it was shown that diligent
search was made to discover its whereabouts but without avail. This A court, where neither of the spouses is domiciled, and to which one
was sufficient to justify the introduction of secondary evidence or both of them may resort merely for the purpose of obtaining a
concerning its contents; and the testimony of the priest showed that divorce, has no jurisdiction to determine their matrimonial status;
the fact of recognition was therein stated. Furthermore, the and a divorce granted by such a court is not entitled to recognition
memorandum in the baptismal record itself constitutes original and elsewhere.
substantive proof of the facts therein.

ISSUE:​ WON the Mory claimants are entitled to the estate


The recognition of Leona as Samuel Bischoff’s daughter occurred
prior to the Civil Code and consequently, her rights as derived from
the recognition must be determined under the law when it existed. HELD: No. With regard to the Mory claimants, their rights principally
Under the law, recognition could be established by proof of acts on depend upon the effect to be given by this court to divorce degree
the part of the parent unequivocally recognizing the status of his granted to Kauffman in Paris. If divorce decree is valid, the
offspring. In other words, tacit recognition was sufficient. subsequent marriage of Leona and Mory is also valid.

ISSUE: WON Felisa Castro (Leona’s mother) was without legal


impediment Since the divorce granted by the French court cannot be recognized,
HELD: Yes. There is no evidence to show that Felisa Castro was it follows that the marriage between Mory and Leona in London
either a single woman or widow at the time of Leona’s birth. In the could not legalize their relations. The claims of the Mory children to
absence of proof to the contrary, however, it must be presumed participate in the estate of Bischoff must be rejected. The right to
that she was a single woman or a widow. inherit is limited to legitimate, legitimated, and acknowledged
natural children.
The contrary presumption would be that Felisa was guilty of adultery
cannot be entertained. If such had been the case, the burden of
proving it would have been upon the persons impugning the ISSUE: What is the effect of the probate of a will upon the rights of
recognition of the child by her father​. forced heirs who did not appear to contest the probate?

ISSUE: WON Leona stands to inherit from the estate of Samuel


Bischoff HELD: The rights of the forced heirs to their legitime are not
HELD: Yes. From the fact that Leona was an acknowledged natural divested by the decree admitting a will to probate—this is regardless
daughter of the decedent, it follows that had she survived him she of the fact that no provision has been made for them in the will, for
the decree of probate is conclusive only with regard to the due The due execution of the will was not established. The only evidence
execution of the will. is to be found in the testimony of the petitioner. There was nothing
to indicate that the will was acknowledged by the testator in the
presence of two competent witnesses, or that these witnesses
Under Sec 753 Code of Civil Procedure, forced heirs cannot be subscribed the will in the presence of the testator and of each other
prejudiced by the failure of a testator to provide for them in his will; as the law of West Virginia requires.
and regardless of the intention of the testator to leave all his It was also necessary for the petitioner to prove that the testator
property to his wife, the will is intrinsically invalid insofar as it would had his domicile in West Virginia and not in the Philippines.
operate to cut off their rights.
4 – Manuela Barretto Gonzalez vs. Augusto Gonzalez
DISPOSITIVE: The order allowing Leontina Mory to participate in the G.R. No. L-37048 | March 7, 1933 | Hull, J.
estate of Samuel Bischoff is reversed; and instead the Kauffman
children will be admitted to share equally in 1/3 of the estate as FACTS:
provided in the decision of Judge Powell dated Nov 14, 1916. In
· Manuela and Augusto are citizens of the Philippines
other respects, the decision of Judge Mariano is affirmed.
and residents of Manila. They were married in Manila
in 1919, and lived together as man and wife until
1926. They voluntarily separated and since that time
have not lived together as man and wife.
· Of this union, 4 children were born who are now 11,
10, 8 and 6. Negotiations between the parties
continued for several months, where it was mutually
agreed to give Manuela support for her and her
3. Hix v. Fluemer, ​supra at 41 children of 500 pesos, and the amount will increase in
G.R. No. 32636 | March 17, 1930 | J. Malcolm case of illness or necessity, and the title of certain
properties to be put in her name.
Doctrine:
· Shortly after this agreement, Augusto left the
The laws of foreign jurisdiction do not prove themselves in the Philippines and moved to Nevada, He secured an
Philippine courts. Such laws must be proven as facts. absolute divorce on the ground of desertion, and was
granted. He then moved to California, and moved
back to the Philippines after 1 year.
Facts:
· On the same date that he secured a divorce, he
A.W. Fluemer, as special administrator of Edward Randolph Hix’s married another Filipino citizen and had 3 children.
estate, appealed the decision of the CFI which denied the probate of He also reduced the support of Manuela and the
Edward’s last will and testament. children and has not made payments fixed in the
divorce as alimony.
Anne Cousins Hix contends that Fluemer does not have the authority
to appeal the CFI decision as a special administrator. · Shortly after his return, Manuela brought an action
Fleumer contends that Hix executed the alleged will in West Virginia, before CFI Manila requesting to ratify the divorce
decree and enfore section 9 of Act No. 2710, which
had his residence in that jurisdiction, and that the laws of West
reads as follows:
Virginia govern.
The decree of divorce shall dissolve the community of
property as soon as such decree becomes final, but shall
ISSUE: not dissolve the bonds of matrimony until one year
Whether the alleged will may be admitted to probate thereafter.

The bonds of matrimony shall not be considered as


RULING:
dissolved with regard to the spouse who, having legitimate
No. Fluemer failed to comply with the law. The Philippine courts are children, has not delivered to each of them or to the
not authorized to take judicial notice laws of the various States of guardian appointed by the court, within said period of one
the American Union. They must be proved as facts. The year, the equivalent of what would have been due to them
requirements of the law were not met. as their legal portion if said spouse had died intestate
● No showing that the book from which an extract was immediately after the dissolution of the community of
property.
taken was printed or published under the authority of the
State of West Virginia. · It was also prayed to deliver to the guardian ​ad litem
● The extract from the law was not attested by the the equivalent of what would have been due to their
certificate of the officer having charge of the original, children as their legal portion from the respective
under the seal of the State of West Virginia. estates. had their parents did intestate on November
● No evidence was introduced to show that the extract from 28, 1927.
the laws of West Virginia was in force at the time the · Further, it is also prayed that the community existing
alleged will was executed.
between Manuela and Augusto be declared dissolved
and Augusto be ordered to render an accounting and
to deliver to Manuela her share of the community he personal views of the writer on the subject of divorce may be. It
property, that Augusto be ordered to pay the is the duty of the courts to enforce the laws of divorce as written by
alimony. the Legislature if they are constitutional. Courts have no right to say
· A guardian ​ad litem was appointed for the minor that such laws are too strict or too liberal.
children, and they appear as intervenors and join
their mother in these proceedings. Litigants by mutual agreement can not compel the courts to approve
· CFI granted judgment in favor of Manuela of their own actions or permit the personal relations of the citizens
of these Islands to be affected by decrees of foreign courts in a
manner which our Government believes is contrary to public order
ISSUE: ​Whether or not the CFI erred in not declaring that paragraph and good morals. Holding the above views it becomes unnecessary
2 of section 9 of the Philippine Divorce Law, is unconstitutional, null to discuss the serious constitutional question presented by appellant
and void. in his first assignment of error.

HELD: ​While the parties in this action are in dispute over financial The judgment of the Court of First Instance of the City of Manila
matters, they are in unity in trying to secure the courts of this must therefore be reversed and defendant absolved from the
jurisdiction to recognize and approve of the Reno divorce. On the demands made against him in this action.
record here presented, this can not be done. The public policy in this
jurisdiction on the question of divorce is clearly set forth in Act No.
2710, 5. Arca v. Javier, 50 O.G. 3538 (1954)
Facts:
The entire conduct of the parties from the time of their separation ● Salud Arca and Alfredo Javier were married in Manila.
until the case was submitted to this court, in which they all prayed They begot one child named Alfredo Javier Junior. In 1938,
that the Reno divorce be ratified and confirmed, clearly indicates a Alfredo left for the US to serve the US Navy. Salud stayed
purpose to circumvent the laws of the Philippine Islands regarding with her in-laws but she did not get along with them so
divorce and to secure for themselves a change of status for reasons she left her in-laws’s house.
and under conditions not authorized by our law. At all times the ● This caused a strain on the relationship of Salud and
matrimonial domicile of this couple has been within the Philippine Alfredo. Alfredo then filed for divorce in Mobile County,
Islands and the residence acquired in the State of Nevada by the State of Alabama. Salud answered the complaint for
husband of the purpose of securing a divorce was not a bona fide divorce and claimed that Alfredo cannot file the complaint
residence and did not confer jurisdiction upon the Court of that since he was not a resident of that state 12 months before
State to dissolve the bonds if matrimony in which he had entered in the filing of the petition and that Alfredo was a resident of
1919. While the decisions of this court in refusing to recognize the Cavite.
validity of foreign divorce has usually been expressed in the negative ● Salud also alleged that it was not true that it was her
and have been based upon lack of matrimonial domicile or fraud or action of leaving the home of her in laws that is a sufficient
collusion, we have not overlooked the provisions of the Civil Code ground to file a divorce on the ground of desertion. It was
now in force in these Islands. Alfredo who had to leave since he was already enlisted in
the US Naval Force.
● The Circuit Court of Mobile County, Alabama granted the
Article 9 thereof reads as follows: complaint for divorce. Alfredo then married Thelma
Francis who was an american citizen but Thelma later on
obtained a divorce.
The laws relating to family rights and duties, or to the ● Alfredo returned to the Philippines and then he married
status, condition and legal capacity or persons, are binding Maria Odvina. Salud then filed a complaint for bigamy
upon Spaniards even though they reside in a foreign against Alfredo. The CFI acquitted Alfredo since it
country. recognized the divorce decree obtained by Alfredo from
Mobile County.

And article 11, the last part of which reads:


Issue: ​WON the divorce decree obtained by Alfredo have a valid
. . . the prohibitive laws concerning persons, their acts and
effect in the Philippines
their property, and those intended to promote public
order and good morals, shall nor be rendered without
effect by any foreign laws or judgments or by anything Held: ​No.
done or any agreements entered into a foreign country.
The foreign court was not able to obtain jurisdiction over the person
of Salud. It is true that Salud R. Arca filed an answer in the divorce
It is therefore a serious question whether any foreign divorce case instituted at the Mobile County in view of the summons served
relating to citizens of the Philippine Islands, will be recognized in this upon her in this jurisdiction, but this action cannot be interpreted as
jurisdiction, except it be for a cause, and under conditions for which placing her under the jurisdiction of the court because its only
the courts of Philippine Islands would grant a divorce. The lower purpose was to impugn the claim of appellant that his domicile or
court in granting relief as prayed for frankly stated that the securing legal residence at that time was Mobile County, and to show that
of the divorce, the contracting of another marriage and the bringing the ground of desertion imputed to her was baseless and false. Such
into the world of innocent children brings about such a condition answer should be considered as a special appearance the purpose of
that the court must grant relief. The hardships of the existing which is to impugn the jurisdiction of the court over the case.
divorce laws of the Philippine Islands are well known to the
members of the Legislature. It is of no moment in this litigation what
It cannot be said that the Mobile County Court of Alabama had
Issue: W/N petitioner presented sufficient proof to support fact of
acquired jurisdiction over the case for the simple reason that at the
her marriage and can thus be admitted as non-quota immigrant in
time it was filed appellant's legal residence was then in the
the country?
Philippines. He could not have acquired legal residence or domicile
at Mobile County when he moved to that place in 1938 because at
that time he was still in the service of the U.S. Navy and merely Held:
rented a room where he used to stay during his occasional shore
leave for shift duty. That he never intended to live there No. A lot of discrepancies were found in the statements made by
permanently is shown by the fact that after his marriage to Thelma petitioner and her alleged husband in the investigations conducted
Francis in 1941, he moved to New York where he bought a house by the immigration authorities. Also, the only basis in support of
and a lot, and after his divorce from Thelma in 1949 and his petitioner’s claim that she is Blas’ wife is a mass of oral and
retirement from the U.S. Navy, he returned to the Philippines and documentary evidence bereft of substantial proof of husband-wife
married Maria Odvina of Naic, Cavite, where he lived ever since. It relationship.
may therefore be said that appellant went to Mobile County, not
with the intention of permanently residing there, or of considering Article 15 of our new Civil Code also provides that ​laws relating to
that place as his permanent abode, but for the sole purpose of family rights or to the status of persons are binding upon citizens
obtaining divorce from his wife. Such residence is not sufficient to of the Philippines, even though living abroad, and it is well-known
confer jurisdiction on the court. that in 1929 in order that a ​marriage celebrated in the Philippines
may be valid it must be solemnized either by a judge of any court
Private International Law prohibits the extension of a foreign inferior to the Supreme Court, a justice of the peace, or a priest or
judgment, or the law affecting the same, if it is contrary to the law minister of the gospel of any denomination duly registered in the
or fundamental policy of the State of the forum. It is also in keeping Philippine Library and Museum (Public Act 3412, Section 2). Even if
with our concept or moral values which has always looked upon we assume, therefore, that the marriage of petitioner to Perfecto
marriage as an institution. And such concept has actually crystallized Blas before a village leader is valid in China, the same is not one of
in a more tangible manner when in the new Civil Code our people, those authorized in our country.
through Congress, decided to eliminate altogether our law relative
to divorce. Because of such concept we cannot but react adversely But it may be contended that under Section 4 of General orders No.
to any attempt to extend here the effect of a decree which is not in 68, as reproduced in Section 19 of Act No. 3613, which is now ​Article
consonance with our customs, morals, and traditions. 71 of our new Civil Code, a marriage contracted outside of the
Philippines which is valid under the law of the country in which it
was celebrated is also valid in the Philippines. But no validity can be
6. Wong Woo Yui v. Vivo, 13 SCRA 552 (1965) given to this contention because no proof was presented relative to
Facts: the law of marriage in China. Such being the case, we should apply
the general rule that in the absence of proof of the law of a foreign
1. ​The Board of Special Inquiry No. 3 rendered a decision country it should be presumed that it is the same as our own.
finding petitioner to be legally married to Perfecto Blas and
admitting her into the country as a non-quota immigrant, which Since our law only recognizes a marriage celebrated before any of
was later on affirmed by the Board of Commissioners. the officers mentioned therein, and a village leader is not one of
2. However, the same Board, composed of a new set of them, it is clear that petitioner’s marriage, even if true, cannot be
members, reversed BSI No. 3 and ordered petitioner to be recognized in this jurisdiction.
excluded from the country.
3. ​Petitioner filed a motion for new trial but the same was Decision appealed from reversed.
denied for lack of merit. She then filed the instant petition for
mandamus with preliminary injunction (considered as 7. Tenchavez v. Escano, G.R. No. L-19671 (29 November 1965)
certiorari) before the Manila CFI.
4. After the respondents filed their answer and the parties 15 SCRA 355
submitted a written stipulation of facts, the court ​a quo
declared valid the original decision and restrained respondents FACTS:
from excluding petitioner from the country. Respondents 27 years old Vicenta Escano who belong to a prominent Filipino
interposed the present appeal. Family of Spanish ancestry got married on Feburary 24, 1948 with
5. ​It appears from the BSI proceeding that petitioner declared Pastor Tenchavez, 32 years old engineer, and ex-army officer
that she came to the Philippines in 1961 for the first time to before Catholic chaplain Lt. Moises Lavares. The marriage was a
join her husband Perfecto Blas to whom she was married in culmination of the love affair of the couple and was duly registered
Chingkang, China on January 15, 1929; ​that their marriage was in the local civil registry. A certain Pacita Noel came to be their
celebrated by one Chua Tio, a village leader​; that the new set match-maker and go-between who had an amorous relationship
of Board of Commissioners found that petitioner’s claim was with Tenchavez as written by a San Carlos college student where
without basis, it appearing that in the entry proceedings of she and Vicenta are studying. Vicenta and Pastor are supposed to
Perfecto Blas had on January 23, 1947 he declared that he first renew their vows/ marriage in a church as suggested by Vicenta’s
visited China in 1935 and married petitioner in 1936, it could parents. However after translating the said letter to Vicenta’s dad
not possibly sustain her claim that she married Perfecto Blas in , he disagreed for a new marriage. Vicenta continued leaving with
1929; that in an affidavit dated August 9, 1962 Perfecto Blas her parents in Cebu while Pastor went back to work in Manila.
claimed that he went to China in 1929, 1935 and 1941,
although in his re-entry declaration he admitted that he first Vicenta applied for a passport indicating that she was single and
went to China in 1935, then in 1937, then in 1939, and lastly in when it was approved she left for the United States and filed a
1941; and that Perfecto Blas in the same affidavit likewise complaint for divorce against Pastor which was later on approved
claimed that he first went to China when he was merely four and issued by the Second Judicial Court of the State of Nevada.
years old so that computed from his date of birth in 1908 it She then sought for the annulment of her marriage to the
must have been in 1912. Archbishop of Cebu. Vicenta married Russell Leo Moran, an
American, in Nevada and has begotten children. She acquired community property, upon complete separation of property, or
citizenship on August 8, 1958. Petitioner filed a complaint against upon any other regime. The pivotal fact in this case is the Nevada
Vicenta and her parents whom he alleged to have dissuaded divorce of the parties.
Vicenta from joining her husband. ISSUE: ​Whether the Nevada divorce is valid
HELD: Yes. There can be no question as to the validity of the Nevada
ISSUE: Whether the divorce sought by Vicenta Escano is valid and divorce in any of the States of the US. The decree is binding on
binding upon courts of the Philippines. private respondent as an American citizen. Owing to the nationality
principle embodied in Article 15 of the Civil Code, only Philippine
nationals are covered by the policy against absolute divorces the
HELD: same being considered contrary to our concept of public policy and
Civil Code of the Philippines does not admit divorce. Philippine morality. However, aliens may obtain divorces abroad, which may
courts cannot give recognition on foreign decrees of absolute be recognized in the Philippines, provided they are valid according
divorce between Filipino citizens because it would be a violation of to their national law. The divorce is likewise valid as to the
the Civil Code. Such grant would arise to discrimination in favor of petitioner.
rich citizens who can afford divorce in foreign countries. The As such, pursuant to his national law, he is no longer the husband of
adulterous relationship of Escano with her American husband is
the petitioner. He has no standing to sue as husband of the
enough grounds for the legal separation prayed by Tenchavez. In
petitioner over their conjugal assets. He is estopped by his own
the eyes of Philippine laws, Tenchavez and Escano are still married.
representation before his own country's court from asserting that
A foreign divorce between Filipinos sought and decreed is not
right to exercise control over their conjugal assets.
entitled to recognition neither is the marriage of the divorcee
entitled to validity in the Philippines. Thus, the desertion and To maintain, as private respondent does, that, under our laws,
securing of an invalid divorce decree by one spouse entitled the petitioner has to be considered still married to private respondent
other for damages. and still subject to a wife's obligations under Article 109 of the Civil
Code cannot be just. Petitioner should not be obliged to live
WHEREFORE, the decision under appeal is hereby modified as together with, observe respect and fidelity, and render support to
follows; private respondent. The latter should not continue to be one of her
(1) Adjudging plaintiff-appellant Pastor Tenchavez entitled to a heirs with possible rights to conjugal property. She should not be
decree of legal separation from defendant Vicenta F. Escaño; discriminated against in her own country if the ends of justice are to
(2) Sentencing defendant-appellee Vicenta Escaño to pay be served.
plaintiff-appellant Tenchavez the amount of P25,000 for damages Petition Granted.
and attorneys' fees;
(3) Sentencing appellant Pastor Tenchavez to pay the appellee, Note: This case was decided before August 3, 1988 when the
Mamerto Escaño and the estate of his wife, the deceased Mena Family Code took effect. It became the basis for the codification of
Escaño, P5,000 by way of damages and attorneys' fees. Article 26 (2) of the Family Code.

8. Van Dorn v. Romillo, 139 SCRA 139 (1985) 9. De Leon v. CA, 186 SCRA 347 (1990)
FACTS: ​Petitioner is a citizen of the Philippines while private SYLVIA LICHAUCO DE LEON,​ ​petitioner,
respondent, RIchard Upton, is a citizen of the United States; that vs.
they were married in Hongkong in 1972; that, after the marriage,
they established their residence in the Philippines; that they begot THE HON. COURT OF APPEALS, MACARIA DE LEON AND JOSE
two children born on April 4, 1973 and December 18, 1975, VICENTE DE LEON,​ ​respondents.
respectively; that the parties were divorced in Nevada, United G.R. No. 80965| June 6, 1990 | MEDIALDEA, J.
States, in 1982; and that petitioner has re-married also in Nevada,
this time to Theodore Van Dorn. Topic: Marriage
A suit against petitioner was filed on June 8, 1983, stating that FACTS:
petitioner’s business in Ermita Manila, the Galleon Shop, is a ● 1969: Private respondent Jose Vicente De Leon and
conjugal property with Upton and prayed therein that Alice be
petitioner Sylvia Lichauco De Leon were united in wedlock
ordered to render an accounting of the business and he be declared
before the Municipal Mayor of Binangonan, Rizal.
as the administrator of the said property.
● 1971: A child named Susana L. De Leon was born from this
Petitioner contends that respondent is estopped from laying claim union.
on the alleged conjugal property because of the representation he ● 1972: A de facto separation between the spouses occured
made in the divorce proceedings before the American Court that due to irreconcilable marital differences, with Sylvia
they had no community of property; that the Galleon Shop was not leaving the conjugal home.
established through conjugal funds, and that respondent's claim is ● 1973: Sylvia went to the United States where she obtained
barred by prior judgment. American citizenship. Same year, she field for divorce as
For his part, respondent avers that the Divorce Decree issued by the well as claims for support and distribution of properties
before the Superior Court of California.
Nevada Court cannot prevail over the prohibitive laws of the
○ However, that since Jose Vicente was then a
Philippines and its declared national policy; that the acts and
Philippine resident and did not have any assets
declaration of a foreign Court cannot, especially if the same is
in the US, Sylvia chose to hold in abeyance the
contrary to public policy, divest Philippine Courts of jurisdiction to
divorce proceedings, and rather concentrated
entertain matters within its jurisdiction.
her efforts to obtain property settlements with
For the resolution of this case, it is not necessary to determine Jose Vicente in the Philippines.
whether the property relations between petitioner and private
respondent, after their marriage, were upon absolute or relative
● 1977: Sylvia succeeded in entering into a · Article 1306 of the New Civil Code provides:
Letter-Agreement with her mother-in-law, private
respondent Macaria De Leon Art. 1306. The contracting parties may establish such stipulations,
○ Letter was a contractual undertaking in clauses, terms, and conditions as they may deem convenient,
consideration for a peaceful and amicable provided they are not contrary to law, morals, good customs, public
termination of relations of the spouses, whereby order or public policy.
it obligates Jose Vicente to adjudicate certain · If the stipulation is contrary to law, morals or public policy, the
properties and sum of money to petitioner Sylvia
contract is void and inexistent from the beginning:
Lichauco De Leon. It also stated the obligations
of wife Sylvia Art. 1409. The following contracts are inexistent and void from the
○ Par. 4 of letter states: beginning:
4.) This contract is intended to be applicable both in the Republic of Those whose cause, object or purpose is contrary to law, morals,
the Philippines and in the United States of America. It is agreed that good customs, public order or public policy;
this will constitute an actionable document in both jurisdictions and xxx xxx xxx
the parties herein waive their right to object to the use of this
document in the event a legal issue should arise relating to the (7) Those expressly prohibited or declared void by law.
validity of this document. In the event of a dispute, this letter is · These contracts cannot be ratified. Neither can the right to set
subject to interpretation under the laws of California, U.S.A
up the defense of illegality be waived. But marriage is not a
● In the same year, Sylvia and Jose Vicente filed before the mere contract but a sacred social institution. Thus, Art. 52 of
then Court of First Instance of Rizal a joint petition for the Civil Code provides:
judicial approval of dissolution of their conjugal Art. 52. Marriage is not a mere contract but an inviolable social
partnership
institution. Its nature, consequences and incidents are governed by
● 1980: After ex-parte hearings, the trial court issued an
law and not subject to stipulations.​..
Order approving the petition
● Jose Vicente moved for a reconsideration of the order · From the foregoing provisions of the New Civil Code, this court
alleging that Sylvia made a verbal reformation of the is of the considered opinion and so holds that intervenor's
petition as there was no such agreement for the payment undertaking under Exhibit 'E' (letter agreement) premised on
of monthly support to commence from the alleged date of the termination of marital relationship is not only contrary to
separation in 1973 and that there was no notice given to law but contrary to Filipino morals and public Policy. As such,
him that Sylvia would attempt verbal reformation of the any agreement or obligations based on such unlawful
agreement contained in the joint petition consideration and which is contrary to public policy should be
○ While pending resolution of MR, Macaria filed a deemed null and void.
motion for leave to intervene alleging that she is · Additionally, Article 191 of the Civil Case contemplates
the owner of the properties involved in the case
properties belonging to the spouses and not those belonging to
and assailed the validity and legality of the
a third party, who, in the case at bar., is Macaria. In the petition
Letter-Agreement
for the dissolution of the conjugal partnership, it was made to
● Trial Court: declared null and void the letter agreement
appear that the said properties are conjugal in nature.
and affirmed the dissolution of the conjugal partnership of
However, Macaria was able to prove that the questioned
the spouses
properties are owned by her.
● CA: affirmed
● On appeal, Sylvia insists that the consideration for her [Other Discussions]
execution of the Letter-Agreement was the termination of · Assuming arguendo that the consideration of the
property relations with her husband. On the other hand,
Letter-Agreement was the termination of property relations, he
Macaria and Jose Vicente assert that the consideration
agreement nevertheless is void because it contravenes the
was the termination of marital relationship.
following provisions of the Civil Code:
Art. 221. The following shall be void and of no effect:
ISSUES:
(1) Any contract for personal separation between husband and
(1) WON the Letter-Agreement is valid- NO wife;
(2) Every extra-judicial agreement, during marriage, for the
HELD: dissolution of the conjugal partnership of gains or of the absolute
community of property between husband and wife;
(1) YES; Petition Denied and decision of CA Affirmed.
DISPOSITIVE:
· This Court finds and holds that the cause or consideration for
ACCORDINGLY, the petition is hereby DENIED. The decision of the
the intervenor Macaria De Leon in having executed Letter
respondent Court of Appeals dated June 30, 1987 and its resolution
Agreement was the termination of the marital relationship
dated November 24, 1987 are AFFIRMED.
between her son Jose Vicente De Leon and Sylvia Lichauco de
Leon.
· In page two of the letter agreement, the parties contemplated 10. HERALD BLACK DACASIN vs SHARON DEL MUNDO DACASIN
not only to agree to a judicial separation of property of the GR No. 168785| February 5, 2010 | Carpio, J. | I.E.B.Z.
spouses but likewise to continue with divorce proceedings. If
taken with the apparently ambiguous provisions regarding
termination of 'relations', ​the parties clearly contemplated not
only the termination of property relationship but likewise of DOCTRINE:
marital relationship in its entirety.
Separated parents cannot contract away the provisions of the Family Petitioner's suit seeks the enforcement not of the "various
Code on the maternal custody of children below seven years. provisions" of the divorce decree but of the post-divorce Agreement
on joint child custody. Thus, the action lies beyond the zone of the
Illinois court's so-called "retained jurisdiction."
FACTS:
On April 1994, petitioner and respondent got married here in the
Philippines. The following year respondent got pregnant and gave OTHER ISSUE:
birth to a baby girl whom they named Stephanie. Whether the agreement or contract is valid?
In June of 1999 respondent sought and obtained from the Illinois
Court a divorce decree against petitioner. In its ruling, the Illinois HELD:
court dissolved the marriage of petitioner and respondent, awarded
to respondent sole custody of Stephanie and retained jurisdiction NO. The trial court cannot enforce the Agreement which is contrary
over the case for enforcement purposes. to law.

On January 28 2002, petitioner and respondent executed in Manila a Under Art. 213 of the Civil Code of the Philippines, it states that “no
contract (Agreement) for the joint custody of Stephanie. Two years child under seven years of age shall be separated from the mother.”
after, petitioner sued respondent in the Regional Trial Court of And at the time that the Dacasins entered into the agreement,
Makati City. Petitioner claimed that respondent exercised sole Stephanie was still below seven-years-old.
custody over Stephanie. Respondent sought the dismissal of the Clearly then, the Agreement's object to establish a post-divorce joint
complaint due to lack of jurisdiction, since Illinois Court hold the custody regime between respondent and petitioner over their child
jurisdiction in enforcing the divorce decree. under seven years old contravenes Philippine law.

RTC: Also, at the time of the writing of this SC decision, Stephanie is


The RTC sustained Sharon’s motion and dismissed Herald’s already 15 years of age. Hence, removing the case outside of the
complaint for lack of jurisdiction. ambit of the mandatory maternal custody regime under Article 213
and bringing it within coverage of the default standard on child
The trial court held that: (1) it is precluded from taking cognizance custody proceedings — the best interest of the child.
over the suit considering the Illinois court's retention of jurisdiction
to enforce its divorce decree, including its order awarding sole The SC suggests that the Dacasins take advantage of the RTC’s
custody of Stephanie to respondent; (2) the divorce decree is jurisdiction to decide as to who has legal custody of Stephanie.
binding on petitioner following the "nationality rule" prevailing in
this jurisdiction; 5 and (3) the Agreement is void for
DISPOSITIVE:
contravening Article 2035, paragraph 5 of the Civil Code 6
WHEREFORE, we REVERSE the Orders dated 1 March 2005 and 23
prohibiting compromise agreements on jurisdiction. June 2005 of the Regional Trial Court of Makati City, Branch 60. The
case is REMANDED for further proceedings consistent with this
ruling.

RELEVANT ISSUE: 11. Yao Kee, et al. v. Sy-Gonzales, G.R. No. L-55960 November 24,
Whether the Trial Court have the jurisdiction over the case? 1988

Doctrine: Well-established in this jurisdiction is the principle that


HELD:
Philippine courts cannot take judicial notice of foreign laws. They
YES. The trial court has jurisdiction to entertain petitioner's suit but must be alleged and proved as any other fact.
not to enforce the Agreement which is void.
At the time petitioner filed his suit in the trial court, ​statutory law Facts:
vests on Regional Trial Courts exclusive original jurisdiction over Sy Kiat, a Chinese national, died in Caloocan City where he was then
civil actions incapable of pecuniary estimation. An action for residing, leaving behind real and personal properties here in the
specific performance, such as petitioner's suit to enforce the Philippines.
Agreement on joint child custody, belongs to this species of actions. Aida Sy-Gonzales together with her siblings, in said petition alleged
Thus, jurisdiction-wise, petitioner went to the right court​. among others that, they are the children of the deceased with
Asuncion Gillego.
The Illinois Court did not strip the RTC of its jurisdiction over the
They do not recognize Sy Kiat’s marriage to Yao Kee nor the filiation
case. What the Illinois court retained was "jurisdiction for the
of her children to him; and they nominate Aida Sy-Gonzales for
purpose of enforcing all and sundry the various provisions of its appointment as administratrix of the intestate estate of the
Judgment for Dissolution." deceased.
The petition was opposed by Yao Kee together with his children No.
alleged that she is the lawful wife of Sy Kiat whom he married on in These evidence may very well prove the fact of marriage between
China and has legitimate children. Sze Sook Wah is the eldest among
Yao Kee and Sy Kiat. However, the same do not suffice to establish
them and is competent to become the administratrix of the estate of
the validity of said marriage in accordance with Chinese law or
Sy Kiat.
custom.
After hearing, the probate court held if favor of the oppositors and
Custom is defined as "a rule of conduct formed by repetition of acts,
appointed Sze Sook Wah as the administratrix of the intestate estate
uniformly observed (practiced) as a social rule, legally binding and
of the deceased.
obligatory"
On appeal the Court of Appeals rendered a decision modifying that
On this score the Court had occasion to state that "a local custom
of the probate court
as a source of right can not be considered by a court of justice
(1) Declaring petitioners Aida Sy-Gonzales, Manuel Sy, Teresita unless such custom is properly established by competent evidence
Sy-Bernabe and Rodolfo Sy acknowledged natural children of the like any other fact"
deceased Sy Kiat with Asuncion Gillego, an unmarried woman with Construing this provision of law the Court has held that to establish
whom he lived as husband and wife without benefit of marriage for
a valid foreign marriage two things must be proven, namely: (1) the
many years.chanrob1es virtual 1aw library
existence of the foreign law as a question of fact; and (2) the
(2) Declaring oppositors Sze Sook Wah, Sze Lai Chu and Sze Chun alleged foreign marriage by convincing evidence
Yen, the acknowledged natural children of the deceased Sy Kiat with In proving a foreign law the procedure is provided in the Rules of
his Chinese wife Yao Kee, also known as Yui Yip, since the legality of
Court. With respect to an unwritten foreign law, ​Rule 130 section 45
the alleged marriage of Sy Kiat to Yao Kee in China had not been
states that:
proven to be valid to the laws of the Chinese People’s Republic of
China. SEC. 45. Unwritten law. — The oral testimony of witnesses, skilled
Sze Sook Wah was appointed by the lower court of as judicial therein, is admissible as evidence of the unwritten law of a foreign
country, as are also printed and published books of reports of
administratrix of the estate of the deceased.
decisions of the courts of the foreign country, if proved to be
From said decision both parties moved for partial reconsideration, commonly admitted in such courts.
which was however denied by respondent court. They thus Proof of a written foreign law, on the other hand, is provided for
interposed their respective appeals to this Court.
under Rule 132 section 25,

Issues:
SEC. 25. Proof of public or official record. — An official record or an
1st Issue: entry therein, when admissible for any purpose, may be evidenced
Whether or not THE MARRIAGE OF SY KIAT TO YAO YEE has BEEN by an official publication thereof or by a copy attested by the
officer having the legal custody of the record, or by his deputy, and
PROVEN VALID IN ACCORDANCE WITH LAWS OF THE PEOPLE’S
accompanied, if the record is not kept in the Philippines, with a
REPUBLIC OF CHINA.
certificate that such officer has the custody. If the office in which
the record is kept is in a foreign country, the certificate may be
❖ Petitioners argue that the marriage of Sy Kiat to Yao Kee in made by a secretary of embassy or legation, consul general, consul,
vice consul, or consular agent or by any officer in the foreign
accordance with Chinese law and custom was conclusively
service of the Philippines stationed in the foreign country in which
proven.
the record is kept and authenticated by the seal of his office.
The Court has interpreted section 25 to include competent evidence
❖ Yao Kee testified that she was married to Sy Kiat in like the testimony of a witness to prove the existence of a written
Fookien, China ​that she does not have a marriage foreign law
certificate because the practice during that time was for In the case at bar ​petitioners did not present any competent
elders to agree upon the betrothal of their children, and
evidence relative to the law and custom of China on marriage. The
in her case, her elder brother was the one who
testimonies of Yao and Gan Ching cannot be considered as proof of
contracted or entered into an agreement with the
China’s law or custom on marriage not only because they are
parents of her husband; that the agreement was that she
self-serving evidence, but more importantly, there is no showing
and Sy Kiat would be married.
that they are competent to testify on the subject matter. For
❖ She and Sy Kiat were married for 46 years already and the
failure to prove the foreign law or custom, and consequently, the
document was left in China and she doubt if that
validity of the marriage in accordance with said law or custom, the
document can still be found now.
marriage between Yao Kee and Sy Kiat cannot be recognized in this
❖ They lived immediately together as husband and wife
jurisdiction.
❖ The certification issued in Manila by the Embassy of the
People’s Republic of China to the effect that "according to Petitioners have not shown any proof that the Chinese law or
the information available at the Embassy Mr. Sy Kiat a custom obtaining at the time the Sy Joc Lieng marriage was
Chinese national and Mrs. Yao Kee alias Yui Yip also celebrated in 1847 was still the law when the alleged marriage of Sy
Chinese were married in Fukien, the People’s Republic of Kiat to Yao Kee took place in 1931 or eighty-four (84) years later.
China"
❖ They presented a witness, one Li Ung Bieng, to prove that
matrimonial letters mutually exchanged by the contracting 2nd Issue:
parties constitute the essential requisite for a marriage to II. Whether or not RESPONDENT COURT OF APPEALS GRAVELY
be considered duly solemnized in China.
ERRED IN DECLARING AIDA SY-GONZALES, MANUEL SY, TERESITA
Ruling:
SY-BERNABE AND RODOLFO SY AS NATURAL CHILDREN OF SY KIAT ● Judge Velez treated the petition as an action for
WITH ASUNCION GILLEGO custody of a dead body WITHOUT the Vargases’
filing of a separate civil action and WITHOUT
dismissing the first petition for habeas corpus
No that was filed; granted the Vargases the custody
As petitioners failed to establish the marriage of Yao Kee with Sy Kiat of Vitaliana’s dead body.
according to the laws of China, they cannot be accorded the status Petition for review with application for a TRO and/or PI was filed by
of legitimate children but only that of acknowledged natural Eugenio and was followed by another petition which he filed as well,
children. Petitioners are natural children, it appearing that at the hence, the consolidation of cases.
time of their conception Yao Kee and Sy Kiat were not disqualified ● Eugenio failed to substantiate his claim over jthe
by any impediment to marry one another
custody of Vitaliana’s dead body.
Private respondents on the other hand are also the deceased’s
ISSUES:
acknowledged natural children with Asuncion Gillego, a Filipina with
whom he lived for twenty-five (25) years with out the benefit of 1. Whether RTC has jurisdiction over proceedings concerning
marriage. They have in their favor their father’s acknowledgment, an action for custody/possession/authority to bury the
evidenced by a compromise agreement entered into by and deceased/recovery of the dead;
between their parents and approved by the Court of First Instance 2. Whether Eugenio, as a common law spouse, has the right
wherein Sy Kiat not only acknowledged them as his children by to have custody of Vitaliana’s dead body and bury the
Asuncion Gillego but likewise made provisions for their support and same.
future inheritance.

HELD:
Accordingly, this Court finds no reversible error committed by
1. YES​, ​Section 19, Batas Pambansa Blg. 129 provides for the
respondent court.
exclusive original jurisdiction of the Regional Trial Courts
over civil cases. Under Sec. 2, Rule 102 of the Rules of
WHEREFORE, the decision of the Court of Appeals is hereby Court, the writ of ​habeas corpus may be granted by a
Court of First Instance (now Regional Trial Court). It is an
AFFIRMED
elementary rule of procedure that what controls is not the
caption of the complaint or petition; but the allegations
12. Eugenio v. Hon. Velez (RTC Judge Cagayan de Oro City), 185 therein determine the nature of the action, and even
without the prayer for a specific remedy, proper relief may
SCRA 435 (1990)
nevertheless be granted by the court if the facts alleged in
FACTS: ​This case stemmed from a petition for habeas corpus filed by the complaint and the evidence introduced so warrant.
Private Respondents Vargases (full blood bros and sissies of Vitaliana 2. NO​, while it is true that our laws do not just brush aside
Vargas) in September 27, 1988, to have her released by Tomas the fact that such common law relationships are present in
Eugenio. The Vargases alleged that Vitaliana was forcibly taken by our society, and that they produce a community of
Eugenio and was locked up in his residence in Misamis Oriental. properties and interests which is governed by law,
authority exists in case law to the effect that such form of
co-ownership requires that the man and woman living
However, Vitaliana had already died back in August 28, 1988, thus, together must not in any way be incapacitated to contract
despite of the issuance of the writ of habeas corpus, it was returned marriage. In any case, Eugenio r has a subsisting marriage
unsatisfied as Eugenio refused to surrender Vitaliana’s body because with another woman, a legal impediment which
a corpse cannot be a subject of habeas corpus proceedings and he disqualified him from even legally marrying Vitaliana.
had already obtained the permits needed for Vitaliana’s burial. Thus, the custody of the dead body of Vitaliana was
● Eugenio then filed an urgent motion to dismiss correctly awarded to her surviving brothers and sisters
(the Vargases).
for lack of jurisdiction for a petition for habeas
corpus only extends to cases of illegal Section 1103 of the Revised Administrative
confinement/detention of a live person. Code provides:
Sec. 1103. Persons charged with duty of burial​.
The Vargases amended their petition and claimed that they were the — The immediate duty of burying the body of a deceased
person, regardless of the ultimate liability for the expense
ones of duty to bury her and as legal custodians and next of kin, they
thereof, shall devolve upon the persons hereinbelow
should have the body of their sister.
specified:
● An exchange of pleadings ensued and a motion
xxx xxx xxx
to dismiss was finally submitted for resolution.
● Judge Velez viewed the coffin in which Vitaliana (b) If the deceased was an unmarried man or
was placed and moved for the denial of woman, or a child, and left any kin, the duty of burial shall
Eugenio’s motion to dismiss and held that since devolve upon the nearest of kin of the deceased, if they be
Vitaliana was already dead on August 28, 1988, adults and within the Philippines and in possession of
but only revealed to the court on September 29, sufficient means to defray the necessary expenses.
1988, it may entertain through the ​allegations in
the body of the petition on the determination as
to who is entitled to the custody of the dead 13. Board of Commissioners v. Dela Rosa, G.R. Nos. 95122-23 (31
body of the late Vitaliana Vargas as well as the May 1991)
burial or interment (Sec. 19, Batas Pambansa
Blg. 129).
Facts order commanding the arrest of respondent William Gatchalian. The
On July 12, 1960, Santiago Gatchalian, grandfather of William latter appeared before Commissioner Domingo on August 20, 1990
Gatchalian, was recognized by the Bureau of Immigration as a native and was released on the same day upon posting P200,000.00 cash
born Filipino citizen following the citizenship of his natural mother, bond.
Marciana Gatchalian . Before the Citizenship Evaluation Board,
Santiago Gatchalian testified that he has five (5) children with his CID contended that the arrest of respondent follows as a matter of
wife Chu Gim Tee, namely: Jose Gatchalian, Gloria Gatchalian, consequence based on the warrant of exclusion issued on July 6,
Francisco Gatchalian, Elena Gatchalian and Benjamin Gatchalian. 1962

On June 27, 1961, William Gatchalian, then a twelve-year old minor, ISSUE​:
arrived in Manila from Hongkong together with Gloria, Francisco, WON the arrest of the respondent is valid
and Johnson, all surnamed Gatchalian. They had with them
Certificates of Registration and Identity issued by the Philippine HELD:
Consulate in Hongkong based on a cablegram bearing the signature
of the then Secretary of Foreign Affairs, Felixberto Serrano, and No, Sec. 37 (a) of Commonwealth Act No. 613, as amended,
sought admission as Filipino citizens. Gloria and Francisco are the otherwise known as the Immigration Act of 1940, reads:
daughter and son, respectively, of Santiago Gatchalian; while Sec. 37. (a) The following aliens shall be arrested upon the warrant
William and Johnson are the sons of Francisco. After investigation, of the Commissioner of Immigration or of any other officer
the Board of Special Inquiry No. 1 rendered a decision admitting designated by him for the purpose and deported upon the warrant
William Gatchalian and his companions as Filipino citizens. As a of the Commissioner of Immigration after a determination by the
consequence thereof, William Gatchalian was issued Identification Board of Commissioner of the existence of the ground for
Certificate by the immigration authorities on August 16, 1961. deportation as charged against the alien.

On January 24, 1962, the then Secretary of Justice issued From a perusal of the above provision, it is clear that in matters of
Memorandum No. 9 setting aside all decisions purporting to have implementing the Immigration Act insofar as deportation of aliens
been rendered by the Board of Commissioners on appeal or on are concerned, the Commissioner of Immigration may issue
review motu proprio of decisions of the Board of Special Inquiry. The warrants of arrest only after a determination by the Board of
same memorandum directed the Board of Commissioners to review Commissioners of the existence of the ground for deportation as
all cases where entry was allowed on the ground that the entrant charged against the alien.
was a Philippine citizen. Among those cases was that of William and In other words, a warrant of arrest issued by the Commissioner of
others. Immigration, to be valid, must be for the sole purpose of executing a
final order of deportation. A warrant of arrest issued by the
On July 6, 1962, the new Board of Commissioners, after a review Commissioner of Immigration for purposes of investigation only, as
motu proprio of the proceedings had in the Board of Special Inquiry, in the case at bar, is null and void for being unconstitutional.
reversed the decision of the latter and ordered the exclusion of,
among others, respondent Gatchalian. A warrant of exclusion also As We held in Qua Chee Gan vs. Deportation Board (supra), "(t)he
dated July 6, 1962 was issued alleging that "the decision of the constitution does not distinguish warrants between a criminal case
Board of Commissioners dated July 6, 1962 . . . has now become and administrative proceedings. And if one suspected of having
final and executory. committed a crime is entitled to a determination of the probable
cause against him, by a judge, why should one suspected of a
Sometime in 1973, respondent Gatchalian, as well as the others violation of an administrative nature deserve less guarantee?" It is
covered by the July 6, 1962 warrant of exclusion, filed a motion for not indispensable that the alleged alien be arrested for purposes of
re- hearing with the Board of Special Inquiry where the deportion investigation. If the purpose of the issuance of the warrant of arrest
case against them was assigned. On March 14, 1973, the Board of is to determine the existence of probable cause, surely, it cannot
Special Inquiry recommended to the then Acting Commissioner pass the test of constitutionality for only judges can issue the same
Victor Nituda the reversal of the July 6, 1962 decision of the then (Sec. 2, Art. III, Constitution).
Board of Commissioners and the recall of the warrants of arrest
issued therein. A reading of the mission order/warrant of arrest issued by the
Commissioner of Immigration, clearly indicates that the same was
On March 15, 1973, Acting Commissioner Nituda issued an order issued only for purposes of investigation of the suspects, William
reaffirming the July 6, 1961 decision of the Board of Special Inquiry Gatchalian included. Paragraphs 1 and 3 of the mission order directs
thereby admitting respondent Gatchalian as a Filipino citizen and the Intelligence Agents/Officers to:
recalled the warrant of arrest issued against him. xxx xxx xxx
1. Make a warrantless arrest under the Rules of Criminal Procedure,
On June 7, 1990, the acting director of the National Bureau of Rule 113, Sec. 5, for violation of the Immigration Act, Sec. 37, para.
Investigation wrote the Secretary of Justice recommending that a; Secs. 45 and 46 Administrative Code;
respondent Gatchalian along with the other applicants covered by xxx xxx xxx
the warrant of exclusion dated July 6, 1962 be charged with violation 3. Deliver the suspect to the Intelligence Division and immediately
of Sec. 37 (a), pars. 1 and 2, in relation to Secs. 45 (c), and (d) and (e) conduct custodial interrogation, after warning the suspect that he
of Commonwealth Act No. 613, as amended, also known as the has a right to remain silent and a right to counsel; . . .
Immigration Act of 1940.
Hence, petitioners' argument that the arrest of respondent was
On August 1, 1990, the Secretary of Justice indorsed the based, ostensibly, on the July 6, 1962 warrant of exclusion has
recommendation of the NBI to the Commissioner of Immigration for obviously no leg to stand on. The mission order/warrant of arrest
investigation and immediate action. made no mention that the same was issued pursuant to a final order
of deportation or warrant of exclusion.
On August 15, 1990, petitioner Commissioner Domingo of the Furthermore, petitioners' position is not enhanced by the fact that
Commission of Immigration and Deportation * issued a mission respondent's arrest came twenty-eight (28) years after the alleged
cause of deportation arose. Section 37 (b) of the Immigration Act much more on respondent William Gatchalian who was then a
states that deportation "shall not be effected . . . unless the arrest in twelve-year old minor. The Fact is, as records indicate, Santiago
the deportation proceedings is made within five (5) years after the was not pressed by the Citizenship InvestigationBoard to prove
cause of deportation arises. the laws of China relating to marriage, having been content
with the testimony of Santiago that the Marriage Certificate was
In the case at bar, petitioners' alleged cause of action and lost or destroyed during the Japanese occupation of China.
deportation against herein respondent arose in 1962. However, the Neither was Francisco Gatchalian testimony subjected to the same
warrant of arrest of respondent was issued by Commissioner scrutiny by the Board of Special Inquiry. Nevertheless, the
Domingo only on August 15, 1990 — 28 long years after. It is clear testimonies of Santiago Gatchalian and Francisco Gatchalian
that petitioners' cause of action has already prescribed and by their before the Philippine consular and immigration authorities
inaction could not now be validly enforced by petitioners against regarding their marriages, birth and relationship to each other are
respondent William Gatchalian. Furthermore, the warrant of not self-serving but are admissible in evidence as statements or
exclusion dated July 6, 1962 was already recalled and the declarations regarding family reputation or tradition in matters of
Identification certificate of respondent, among others, was pedigree (Sec. 34, Rule 130). Furthermore, this salutary rule of
revalidated on March 15, 1973 by the then Acting Commissioner evidence Hnds support in substantive law. Thus, Art. 267 of the Civil
Nituda. Code Provides: "Art. 267. In the absence of a record of birth,
authentic document, Final judgment
Relevant argument of petitioners: Santiago Gatchalian’s or possession of status, legitimate Hliation may be proved by any
(grandfather) marriage with Chiu Gim Tee in China as well as other means allowed by the Rules of Court and special laws."
Francisco’s (father) marriage to Ong Chiu Kiok in China were not
supported by any evidence other than their own self-serving Having declared the assailed themarriages as valid, respondent
testimony nor was there any showing of what laws of China were. William Gatchalian follows the citizenship of his father Francisco,
There being no showing of validity, they cannot be considered valid. a Filipino, as a legitimate child of the latter. Francisco, in turn,
Therefore, citizenship of illegitimate children follow their Chinese is likewise a Filipino being the legitimate child of Santiago
mothers’. Gatchalian who (the latter) is admittedly a Filipino citizen whose
Philippine citizenship was recognized by the Bureau of Immigration
Issue​: In an order dated July 12, 1960. Finally, respondent William
Whether the marriages are valid (YES) Gatchalian belongs to the class of Filipino citizens contemplated
under Sec. 1, Article IV of the Constitution, which provides:
Held​: "Section 1. The following are citizens of the Philippines: "(1)
In Miciano vs. Brimo (50 Phil. 867 [1924]; Lim Those who are citizens of the Philippines at the time of the
and Lim vs. Collector of Customs, 36 Phil. 472; Yam Ka Lim vs. adoption of this Constitution. . . ." This Forecloses any further
Collector of Customs, this Court held that in the absence of evidence question about the Philippine citizenship of respondent William
to the contrary, foreign laws on a particular subject are presumed Gatchalian.
to be the same as those of the Philippines.
In the case at bar, there being no proof of Chinese law relating 14. San Luis vs. San Luis 514 SCRA 294 (2007)
to marriage, there arises the
presumption that it is the same as that of Philippine law. Doctrine​: ​Foreign divorce decree must be proven that they are
validly obtained under foreign laws.
Philippine law, following the lex loci celebrationis, adheres to FACTS:
the rule that a
marriage formally valid where celebrated is valid everywhere. ● Involves settlement of estate of Felicisimo who contracted
Referring to marriages 3 marriages during his lifetime.
contracted abroad, Art. 71 of the Civil Code (now Art. 26 of the 1​st​ marriage is with Sulit but she predeceased him
Family Code) provides that
"(a)ll marriages performed outside of the Philippines in accordance 2​nd marriage is with Corwin celebrated in the US. Corwin
with the laws in force obtained divorce decree which was granted.
in the country where they were performed, and valid there as such, 3​rd​ marriage is with Felicidad celebrated in LA.
shall also be valid in this
country . . ." And any doubt as to the validity of the matrimonial ● With that, Felicidad petitioned for letters of administration
unity and the extent as to of Estate before RTC Makati even though they live in
how far the validity of such marriage may be extended to the Alabang.
consequences of the ● Petitioner, as one of the heirs in 1​st marriage, moved to
coverture is answered by Art. 220 of the Civil Code in this dismiss Felicidad’s petition on the grounds of improper
manner: "In case of doubt, all venue and failure to state cause of action. He argued that
presumptions favor the solidarity of the family. ○ The proceedings should be filed in Laguna since
Thus, every intendment of law or facts leans toward the validity of that was where the decedent last resided and
marriage, the indissolubility of the marriage bonds, the ○ Felicidad has no legal personality because her
legitimacy of children, the community of property during marriage, marriage was invalid, being Felicisimo still
the authority of parents over their children, and the validity of married to Corwin
defense for any member of the family in case of unlawful ● RTC ruled in favor of Felicidad. This decision was
aggression." Bearing in mind the "processual presumption" reinstated by CA.
enunciated in Miciano And other cases, he who asserts that the ISSUES and HELD:
marriage is not valid under our law bears the burden of proof to
present the foreign law. 1. Whether venue is improperly laid - NO
○ Settlement of estate must be made at the place where the
The lack of proof of Chinese law on the matter cannot be blamed on
decedent last resided if he is a resident. Resides mean the
Santiago Gatchalian
actual and physical presence in the place. In this case,
there were sufficient evidence proving that Felicisimo last final divorce decree under his national law prior to his filing the
resided in Alabng before his demise. Among the evidence criminal complaint.
presented are: Hospital billings indicating his address to be
Alabang, Membership on Alabang Villag Assoc and Ayala
Country club, Calling cards of deceased. ISSUE
2. Whether Felicidad has legal capacity to petition for letters of Whether respondent has capacity to sue even if at the time he filed
administration – YES under ROC but it cannot yet be the complaint, he was already divorced petitioner
determined if she has legal capacity by virtue of their
marriage
HELD
○ Even though the 3​rd marriage was celebrated before
Family Code retroactivity must apply to Article 26, which No.
entitled a Filipino spouse to remarry after the alien
obtained divorce decree.
○ A foreign divorce decree is recognized here in the Under Article 344 of the RPC, the crime of adultery, as well as four
Philippines so that the Filipino spouse would not be left other crimes against chastity, cannot be prosecuted except upon a
without remedy or choice. Making therefore the Filipino sworn written complaint filed by the ​offended spouse​, ​and nobody
spouse capacitated to remarry under Philippine laws. else​. ​Such is jurisdiction and not merely a formal requirement.
○ This is pursuant to a non-discrimination principle. And
so, although Articles 15 and 17 of NCC which declares
divorce void under Philippine laws, Article 26 FC and Unlike offenses of seduction, abduction, rape and acts of
jurisprudence provided otherwise. lasciviousness where there is a specific provision on which allows
○ HOWEVER, foreign laws need be proven to attest that other person to file and prosecute, no provision relates to adultery
foreign divorce decree is valid thereunder. Thus, this and concubinage. Thus, only the offended spouse, and no other, is
case is remanded to trial court so that evidence may be authorized by law to initiate the action.
received to that effect.
○ NONETHELESS, even tho divorce of 2​nd marriage not
proven so as to render marriage to Felicidad valid, she is Corollary to this, it is necessarily follows that such initiator must
still an interested person under ROC entitled to issuance have the status, capacity or legal representation to do so at the time
of letters of administration. This is premised on Article of the filing of the criminal action. This is an express rule in civil
148 FC which governs property relations of couples living actions. Lack of legal capacity to sue is a ground for a motion to
together but incapacitated to marry. During their dismiss.
cohabitation, their properties are jointly owned by them.
And so, Felicidad is treated a co-owner of Felicisimo’s
properties. Absence of explicit rule in criminal case does not prevent the
PETITION DENIED application. Understandably, it may not have been found necessary
since criminal actions are generally and fundamentally commenced
by the State, through the People of the Philippines, the offended
15 Pilapil vs. Ibay-Somera party being merely the complaining witness therein. However, in the
so-called “private crimes”, or those which cannot be prosecuted​de
G.R. No. 80116, June 30, 1989 oficio, ​and the present prosecution for adultery is of such genre, the
offended spouse assumes a more predominant role since the right
to commence the action, or to refrain therefrom, is a matter
FACTS exclusively within his power and option.
Petitioner Imelda Pilapil (Filipino) and respondent Erich Geiling
(German) were married in Germany in 1979. Thereafter, marital Article 344 of the RPC thus presupposes that the marital relationship
discord set in, followed by a separation ​de facto ​between them.
is still subsisting at the time of the institution of the criminal action
After about 3 ½ years of marriage, respondent initiated a divorce
for adultery. This is a logical consequence since the ​raison d'etre ​of
proceeding against petition in Germany claiming failure of marriage.
said provision of law would be absent where the supposed offended
This was granted.
party had ceased to be the spouse of the alleged offender at the
time of the filing of the criminal case.
More than five months after the divorce decree, respondent filed 2
complaints for adultery before City Fiscal of Manila alleging that, Case at bar, respondent has no more standing
while still married to him, petitioner had an affair with certain
William Chia in 1982 and Jesus Chua in 1983. Information were later The status of the complainant vis-a-vis the accused must be
filed. determined as of the time the complaint was filed. After a divorce
has been decreed, the innocent spouse no longer has the right to
institute proceedings against the offenders.
Petitioner filed a motion to quash the information for lack of
jurisdiction. Denied.
Here, said divorce and its legal effects may be recognized in the
Philippines insofar as private respondent is concerned in view of the
Hence, this SCA to annul the order of the lower court denying the nationality principle in our civil law on the matter of status of
motion to quash on ground that court is without jurisdiction “to try persons. When said respondent initiated the divorce proceeding, he
and decide the charge of adultery, which is a private offense that obviously knew that there would no longer be a family nor marriage
cannot be prosecuted ​de officio​, since the purported complainant, a vows to protect once a dissolution of the marriage is decreed.
foreigner, does not qualify as an offended spouse having obtained a
private respondent the capacity to remarry. Thus, the present
controversy mainly relates to the award of the custody of their two
Reversed. Quashed. children, Carolynne and Alexandra Kristine, to petitioner.

As a general rule, divorce decrees obtained by foreigners in other


16. Roeher vs. Rodriguez GR No. 142820, June 20, 2003
countries are recognizable in our jurisdiction, but the legal effects
DOCTRINE: ​As a general rule, divorce decrees obtained by foreigners thereof, ​e.g. ​on custody, care and support of the children, must
in other countries are recognizable in our jurisdiction, but the legal still be determined by our courts. Before our courts can give the
effects thereof, ​e.g. ​on custody, care and support of the children, effect of ​res judicata ​to a foreign judgment, such as the award of
must still be determined by our courts. custody to petitioner by the German court, ​it must be shown that
the parties opposed to the judgment had been given ample
opportunity to do ​so on grounds allowed under Rule 39, Section 50
FACTS of the Rules of Court.
● Petitioner Wolfgang O Roehr, a German citizen and Under the Rules of Court, in actions in personam, a foreign judgment
resident of Germany, married private respondent Carmen merely constitutes prima facie evidence of justness of claim of a
Rodriguez, a Filipina, in Hamburg, Germany. party and is subject to proof to the contrary.
● Their marriage was subsequently ratified on Feb. 14, 1981 In the present case, it cannot be said that the private respondent
in Tayasan, Negros Oriental. Out of their union, Carolynne
was given the opportunity to challenge the judgment of the German
and Alexandra Kristine was born.
court so that there is basis for declaring that the judgment as res
● Private respondent filed a petition for declaration of nullity
judicata as regards to the rights of petitioner to have parental
of marriage before RTC Makati.
custody of their 2 children.
● A motion to dismiss was filed by the petitioner, but was
denied by the RTC. Petitioner’s MR was also denied. Upon Furthermore, the proceedings in the German court were summary.
appeal to the CA, the CA denied the petition and Neither does the divorce decree states that she had commented nor
remanded the case to RTC. given her opinion to the Social Services Office. Also, private
● Meanwhile, petitioner (father) obtained a decree of respondent had no counsel to assist her in the saif proceedings.
divorce from CFI of Hamburg-Blankenese. The parental Absent any finding that private respondent is unfit to obtain custody
custody of the children is granted to the father.
of the children, the trial court was correct in setting the issue for
● By virtue of the said decree, petitioner (father) filed a 2nd
hearing to determine the issue of parental custody, care, support
motion to dismiss on the ground that the RTC had no
and education mindful of the best interests of the children. This is in
jurisdiction over the subject matter as the decree of
consonance with the provision in the Child and Youth Welfare Code
divorce had already been promulgated dissolving the
that the child’s welfare is always the paramount consideration in all
marriage of petitioner and respondent.
questions concerning his care and custody.
● Judge Guevarra-Salonga issued an order granting the
motion to dismiss, while the private respondent (mother) On the matter of property relations, the parties admitted in their
filed a motion for partial reconsideration with a prayer to pleadings that there is no property to be accounted for, respondent
determine the issues of custody and distribution of judge has no basis to assert jurisdiction in this case to resolve a
properties between petitioner and respondent. matter no longer deemed in controversy.
● Respondent judge issued an order granting the prayer of In sum, we find that respondent judge may proceed to determine
private respondent to tackle issues of property relations,
the issue regarding the custody of the two children born of the
support, and custody of their children.
union between petitioner and private respondent. Private
● Hence, this petition. Petitioner alleges lack of jurisdiction
respondent erred, however, in claiming cognizance to settle the
on the part of the trial court.
matter of property relations of the parties, which is not at issue.

Issue: W/N a trial court has no jurisdiction due to the fact of the
17. Corpuz vs. Sto. Ytomas (626 SCRA 266 (2010)
divorce decree validly obtained abroad; hence will constitute res
GR No. 186571 | August 11, 2010 | J. Brion | Manao
judicata.
DOCTRINE: An alien spouse may only file a judicial recognition of
Held: No. divorce under Sec. 48, Rule 39 of the Rules of Court. He cannot file
one under Art. 26 of the Family Code.
In ​Garcia v. Recio, Van Dorn v. Romillo, Jr., ​and ​Llorente v. Court of
Appeals, ​we consistently held that ​a divorce obtained abroad by an FACTS:
alien may be recognized in our jurisdiction, provided such decree is ● Gerbert Corpuz was a former Filipino citizen who acquired
valid according to the national law of the foreigner. Canadian citizenship.
Relevant to the present case is ​Pilapil v. Ibay-Somera, ​where this ● Gerbert married Daisylyn Sto. Tomas, a Filipina.
● Due to work and other professional commitments, Gerbert
Court specifically recognized the validity of a divorce obtained by a
left for Canada soon after the wedding.
German citizen in his country, the Federal Republic of Germany. We
● He returned to the Philippines sometime to surprise
held in ​Pilapil ​that a foreign divorce and its legal effects may be
Daisylyn, but was shocked to discover that his wife was
recognized in the Philippines insofar as respondent is concerned in
having an affair with another man.
view of the nationality principle in our civil law on the status of
● Hurt and disappointed, Gerbert returned to Canada and
persons.
filed a petition for divorce.
In this case, the divorce decree issued by the German court dated ● Superior Court of Justice, Canada: Granted the petition.
December 16, 1997 has not been challenged by either of the parties. ● 2 years after, Gerbert has moved on and has found
In fact, save for the issue of parental custody, even the trial court another Filipina to love.
recognized said decree to be valid and binding, thereby endowing
● Desirous of marrying his new Filipina fiancée in the evidence of a right as between the parties and their
Philippines, Gerbert filed a petition for judicial recognition successors in interest by a subsequent title.”
of foreign divorce and/or declaration of marriage as
dissolved ● Direct involvement or being the subject of the foreign
● RTC: Denied the petition. judgment is sufficient to clothe a party with the
○ Explanation: Gerbert was not the proper party to requisite interest to institute an action before our
institute the action for judicial recognition of the courts for the recognition of the foreign judgment. In
foreign divorce decree as he is a naturalized this case, therefore, Gerbert has legal standing to
Canadian citizen. Only the Filipino spouse can institute a judicial recognition for his divorce.
avail of the remedy, under the second paragraph ● For recognition of divorce, however, the divorce must
of Art. 26, par. 2 of the Family Code. be valid according to the national law of the alien. In
this case, Gerbert failed to include a copy of the
RELEVANT ISSUE: ​Whether or not Gerbert may file a petition for Canadian law on divorce.
judicial recognition of divorce:
(1) Under Art. 26, par. 2 of the Family Code? DISPOSITIVE: ​Remanded to RTC to ascertain whether or not the
(2) Under other laws? divorce is consistent with Canadian law.

HELD: 18. Fujiki vs. Marinay GR No. 196049


(1) NO. Only Filipinos may file a petition for judicial
recognition of divorce under Art. 26, par. 2 of the Family G.R. No. 196049 June 26, 2013
Code. MINORU FUJIKI, PETITIONER, vs. ​MARIA PAZ GALELA MARINAY, et.
● The resolution of the issue requires a review of the
al., ​RESPONDENTS.
legislative history and intent behind Art. 26, par. 2.
○ Our family laws do not recognize absolute
divorce between Filipino citizens. ​Divorce is Topic:​ Family Law
possible, however, in marriages between a
Filipino and an alien. Thus, there can be the
absurd situation where the Filipino spouse Doctrine:​ ​All bold in Held.
remains married to the alien spouse who, after
obtaining a divorce, is no longer married to the
Filipino spouse. Facts:
Former President Cory Aquino amended Art. 26, 1. Petitioner Minoru Fujiki (Fujiki) is a Japanese national who
par. 2 of the Family Code to avoid this situation. married respondent Maria Paz Galela Marinay (Marinay) in the
Said provision reads: Philippines. The marriage did not sit well with Fujiki’s parents. Thus,
Fujiki could not bring his wife to Japan where he resides. Eventually,
Where a marriage between a Filipino citizen and they lost contact with each other.
a foreigner is validly celebrated and a divorce is 2. In 2008, Marinay met another Japanese, Shinichi Maekara
thereafter validly obtained abroad by the alien
(Maekara). Without the first marriage being dissolved, Marinay and
spouse capacitating him or her to remarry, the
Maekara were married in Quezon City. Maekara brought Marinay to
Filipino spouse shall likewise have capacity to
Japan. However, Marinay allegedly suffered physical abuse from
remarry under Philippine law.
Maekara. She left Maekara and started to contact Fujiki.
Essentially, it provided the Filipino spouse a 3. Fujiki and Marinay met in Japan and they were able to
substantive right to have his or her marriage to reestablish their relationship. In 2010, Marinay obtained a judgment
the alien spouse considered as dissolved, from a family court in Japan which declared the marriage between
capacitating him or her to remarry. ​The Marinay and Maekara void on the ground of bigamy.
legislative intent is for the benefit of the 4. Fujiki filed a petition for “Judicial Recognition of Foreign
Filipino spouse.
Judgment (or Decree of Absolute Nullity of Marriage).", praying that
● Given the rationale and intent behind Art. 26, par. 2
(1) the Japanese Family Court judgment be recognized; (2) that the
of the Family Code, only the Filipino spouse can
bigamous marriage between Marinay and Maekara be declared void
invoke said Article of the Family Code; the alien
ab initio ​under the Family Code of the PH and (3) to direct the Local
spouse can claim no right under this provision. The
Civil Registrar of Quezon City to annotate the foreign judgment on
RTC was correct in limiting the applicability of the
the Certificate of Marriage between Marinay and Maekara and to
provision to Filipinos.
endorse such annotation to the NSO.
(2) YES. Gerbert may file a petition for judicial recognition of 5. The RTC dismissed the petition, stating that it was in
the divorce under Sec. 48, Rule 39 of the Rules of Court. “gross violation” of Sec. 2[1] and 4[2] of the Rule on Declaration of
● The said Section provides: Absolute Nullity of Marriages and Annulment of Voidable Marriages
(A.M. No. 02-11-10-SC). It ruled that only “the husband or wife”
“SEC. 48. Effect of foreign judgments or final orders. -- (either Maekera or Marinay), can file the petition to declare the
The effect of a judgment or final order of a tribunal of marriage void, and not Fujiki.
a foreign country, having jurisdiction to render the 6. Fujiki moved to reconsider, arguing that A.M. No.
judgment or final order is as follows:
02-11-10-SC does not apply as it only applies to ordinary civil actions
xxx
for declaration of nullity of marriage, and the case he filed is a
(b) In case of a judgment or final order against a
special proceeding. Fujiki also argued that Sec. 2 only applies to void
person, the judgment or final order is presumptive
marriages on the ground of psych incapacity. Applying the rule that
“a petition may only be filed by the husband or wife” in a bigamy
case would be absurd as only the guilty parties would be permitted of the case as if it were a new petition for declaration of nullity of
to sue. marriage. They cannot substitute their judgment on the status,
7. RTC denied the MR, stating that A.M. No. 02-11-10-SC condition and legal capacity of the foreign citizen who is under the
jurisdiction of another state. Thus, Philippine courts can only
applies since the petition filed, in effect, prays for a decree of
recognize the foreign judgment ​as a fact according to the rules of
absolute nullity of marriage. And under such law, Fujiki is considered
evidence.
a “third person” in the proceeding since he was not the husband in
the decree of divorce issue by the Japanese court; hence, he lacks 5. The court merely exercises limited review on foreign
personality to sue/file the petition. The RTC also held that under judgment. Once a foreign judgment is admitted and proven in a
Braza v The City Civil Registrar of Himamaylan City, the trial court Philippine court, it can only be repelled on grounds external to its
has no jurisdiction to nullify marriages in a special proceeding for merits, ​i.e​. , "want of jurisdiction, want of notice to the party,
correction of entry under Rule 108. collusion, fraud, or clear mistake of law or fact."[4]
8. Upon direction of the Court, the SolGen filed its comment 6. There is therefore no reason to disallow Fujiki to simply
and agreed with the petition, arguing that Fujiki, as the spouse of prove as a fact the Japanese Family Court judgment nullifying the
the first marriage, is an injured party who can sue to declare the marriage between Marinay and Maekara on the ground of bigamy.
bigamous marriage void. It contended that the petition to recognize While the Philippines has no divorce law, the Japanese Family Court
the Japanese court’s judgment may be made in a Rule 109 judgment is fully consistent with Philippine public policy, as
proceeding, as its object is precisely to establish a status or right of a bigamous marriages are declared void from the beginning under the
party or a particular fact. It also argued that the law requires the Family Code.
entry in the civil registry of judicial decrees that produce legal
consequences upon a person’s legal capacity and status. Since the
Japanese court’s judgment directly bears on the civil status of a Fujiki has personality to file the petition to recognize the Japanese
Filipino citizen, it should therefore be proven as a fact in a Rule 108 court’s judgement
proceeding. 1. Once the recognition of a foreign judgment only requires
proof of fact of the judgment, it may be made in a special
Issue/s: [​included all the issues since one way or another, it touches proceeding for cancellation or correction of entries in the civil
registry under Rule 108. As correctly noted by the SolGen, "[t]he
upon foreign judgments on divorce]
recognition of the foreign divorce decree may be made in a Rule 108
1. W/N A.M. No. 02-11-10-SC is applicable proceeding itself, as the object of special is precisely to establish the
2. W/N a husband or wife of a prior marriage can file a status or right of a party or a particular fact."
petition to recognize a foreign judgment nullifying a subsequent 2. There is no doubt that the prior spouse has a personal and
marriage between his or her spouse and a foreign citizen on the material interest in maintaining the integrity of the marriage he
ground of bigamy contracted and the property relations arising from it. There is also
3. W/N the RTC can recognize the foreign judgment in a no doubt that he is interested in the cancellation of an entry of a
bigamous marriage in the civil registry, which compromises the
proceeding for cancellation or correction of entries under Rule 108.
public record of his marriage.
3. Article 35(4) of the Family Code, which declares bigamous
Held: No. A.M. No. 02-11-10-SC does not apply in a petition to marriages void from the beginning, is the civil aspect of Article 349
recognize a foreign judgment relating to the status of a marriage of the Revised Penal Code, which penalizes bigamy. Bigamy is a
where on of t he parties is a citizen of a foreign country. Also, the public crime. Thus, anyone can initiate prosecution for bigamy
rule that only husband or wife can file a declaration of nullity or because any citizen has an interest in the prosecution and
annulment of marriage does not apply if the reason is bigamy. prevention of crimes. If anyone can file a criminal action, there is
more reason to confer personality to sue on the husband or the wife
of a subsisting marriage. The prior spouse does not only share in the
A.M No. 02-11-10-SC is not applicable in this case public interest of prosecuting and preventing crimes, he is also
1. For foreign courts to recognize a foreign judgment, the personally interested in the purely civil aspect of protecting his
marriage.
petition only needs to prove the foreign judgment as a fact,
through (1) an official publication or (2) a certification or copy 4. When the right of the spouse to protect his marriage is
attested by the officer who has custody of the judgment. violated, the spouse is clearly an injured party and is therefore
2. To hold that A.M. No. 02-11-10-SC is applicable is absurd interested in the judgment of the suit. Being a real party in
interest, the prior spouse is entitled to sue in order to declare a
as it will litigate the case anew. It will defeat the purpose of
bigamous marriage void.
recognizing foreign judgments, which is to limit repetitive litigation
on claims and issues.
3. However, the effect of a foreign judgment is not The RTC can recognize the petition in this case
automatic. To extend the effect of a foreign judgment in the 1. In ​Braza v. The City Civil Registrar of Himamaylan City,
Philippines, Philippine courts must determine if the foreign
Negros Occidental​, this Court held that a "trial court has no
judgment is consistent with domestic public policy and other
jurisdiction to nullify marriages" in a special proceeding for
mandatory laws. This is the rule of ​lex nationalii ​in private
cancellation or correction of entry under Rule 108 of the Rules of
international law, as embodied by Art. 15 of the Civil Code[3]. Thus,
Court. Thus, the "validity of marriage[ x x x can be questioned only in
the Philippine State may require, for effectivity in the Philippines,
a direct action" to nullify the marriage.
recognition by Philippine courts of a foreign judgment affecting its
citizen, over whom it exercises personal jurisdiction relating to the 2. However, ​Braza ​does not apply as Braza does not involve a
status, condition and legal capacity of such citizen. recognition of a foreign judgment nullifying a bigamous marriage
4. A petition to recognize a foreign judgment declaring a where one of the parties is a citizen of the foreign country.
marriage void does not require relitigation under a Philippine court
3. In this case, there is neither circumvention of the substantive [3] Art. 15. ​"[l]aws relating to family rights and duties, or to the
and procedural safeguards of marriage under Philippine law, nor of status, condition and legal capacity of persons are binding upon
the jurisdiction of Family Courts under R.A. No. 8369. A recognition citizens of the Philippines, even though living abroad."
of a foreign judgment is not an action to nullify a marriage. It is an [4] Sec. 48, Rule 39. Xxx ​"the judgment or final order may be
action for Philippine courts to recognize the effectivity of a foreign
repelled by evidence of a want of jurisdiction, want of notice to the
judgment, which presupposes a case which was already tried and
party, collusion, fraud, or clear mistake of law or fact."
decided under foreign law.
4. Article 26 of the FC confers jurisdiction on Philippine
19. Ramirez Marcaida vs. Aglubat 21 SCRA 1033 (1962)
courts to extend the effect of a foreign divorce decree to a Filipino
G.R. No. L-24006, November 25, 1967
spouse without undergoing trial to determine the validity of the
dissolution of the marriage. The second paragraph of Article 26 of DOCTRINE: ​[Adoption] An adoption created under the law of a
the Family Code provides that "[w]here a marriage between a foreign country is entitled to registration in the corresponding civil
Filipino citizen and a foreigner is validly celebrated and a divorce is register of PH. It is to be understood, however, that the effects of
thereafter validly obtained abroad by the alien spouse capacitating such adoption shall be governed by the laws of this country.
him or her to remarry, the Filipino spouse shall have capacity to
remarry under Philippine law." In ​Republic v. Orbecido​, this Court
recognized the legislative intent of the second paragraph of Article FACTS:
26 which is "to avoid the absurd situation where the Filipino spouse ● Proceedings for adoption started in CFI of Madrid, Spain by
remains married to the alien spouse who, after obtaining a divorce,
Maria Garnier Garreau (84 yrs old), adopting Josefina Ramirez
is no longer married to the Filipino spouse" under the laws of his or
Marcaida (55 yrs old), a PH citizen. Both were residents of
her country.
Madrid, Spain.
5. The principle in the second paragraph of Article 26 of the ● CFI Madrid granted the application for adoption and gave the
Family Code applies because the foreign spouse, after the foreign necessary judicial authority, once the judgment becomes final,
judgment nullifying the marriage, is capacitated to remarry under to execute the corresponding adoption document pursuant to
the laws of his or her country. If the foreign judgment is not Art. 177 of the Civil Code.
recognized in the Philippines, the Filipino spouse will be ○ A deed of adoption was necessary because under Article
discriminated—the foreign spouse can remarry while the Filipino 177 of the Civil Code of Spain, “the grant by the Judge of
spouse cannot remarry. the adoption must be in writing, expressing in it the
6. Notwithstanding Article 26 of the Family Code, Philippine conditions, and shall be registered in the corresponding
Civil Registry”
courts already have jurisdiction to extend the effect of a foreign
● In compliance, the notarial document of adoption, embodying
judgment in the Philippines to the extent that the foreign judgment
the court order of adoption, and where Maria formally adopted
does not contravene domestic public policy. In the recognition of
Josefina, was executed before Notary Public Carrasquedo of
foreign judgments, Philippine courts are incompetent to substitute
Madrid. In that document, Maria Gernier Garreau instituted
their judgment on how a case was decided under foreign law.
Josefina, amongst other conditions as her unique and universal
7. ​For this purpose, ​Philippine courts will only determine (1) heir of all present and future assets, rights and actions.
whether the foreign judgment is inconsistent with an overriding ● This escritura de adopcion (deed of adoption) was
public policy in the Philippines; and (2) whether any alleging party authenticated by Philippine Vice Consul Martinez in Madrid,
is able to prove an extrinsic ground to repel the foreign judgment, who issued the corresponding certificate of authentication.
i.e. want of jurisdiction, want of notice to the party, collusion, ● The Deed of adoption was filed in the Office of the Local Civil
fraud, or clear mistake of law or fact. If there is neither Registrar of Manila. The Registrar, however, refused to register
inconsistency with public policy nor adequate proof to repel the on the ground that under PH law, adoption can only be had
judgment, Philippine courts should, by default, recognize the through judicial proceeding. Since the notarial document of
foreign judgment as part of the comity of nations. adoption is not a judicial proceeding, it is not entitled to
8. Upon recognition of the foreign judgment, this right registration.
● When the Registrar denied the MR, a petition for mandamus
becomes conclusive and the judgment serves as the basis for the
was filed in CFI Manila. CFI dismissed the mandamus, declaring
correction or cancellation of entry in the civil registry. Otherwise,
the escritura de adopcion as not registrable. Hence, petition.
there will be an inconsistency between the recognition of the
effectivity of the foreign judgment and the public records in the
Philippines. Petitioner’s Contention:
1. Act 3753, Sec. 11 provides that a "civil register is established for
recording the civil status of persons, in which shall be entered,"
amongst others, "(g) adoptions."
2. Art. 407 of PH Civil Code provides that "[t]he following shall be
[1] Sec. 2. Sec. 2. Petition for declaration of absolute nullity of void entered in the civil register: (8) adoptions"
marriages. –
(a) ​Who may file​. – A petition for declaration of absolute nullity of
OSG’s Contention:
void marriage may be filed solely by the husband or the wife.
● The case does not come within the purview of Art. 409 of the
Civil Code, and Sec. 11 of Act 3753. These legal provisions refer
[2] Sec. 4. ​Venue​. – The petition shall be filed in the Family Court of to adoptions effected in PH as they speak of adoption which
the province or city where the petitioner or the respondent has shall be registered in the municipality or city where the court
been residing for at least six months prior to the date of filing, or in issuing the adoption decree is functioning.
the case of a non-resident respondent, where he may be found in
the Philippines, at the election of the petitioner. x x x
ISSUE: ​WON CFI Manila erred in declaring that 'escritura de ● Michael Magno Madayag is the legitimate son of Marcelo
Madayag and Zenaida Magno. Due to poverty Michael’s
adopcion' is not registrable in the Philippines ​(YES)
parents was prompted to have him adopted by the Millers.
They executed affidavits giving their irrevocable consent to
HELD: the adoption by Millers.
● Sps. Miller sought to adopt the minor Michael Magno
The aforesaid provisions were incorporated into the statute books Madayag. They filed a petition to adopt Magno.
merely to give effect to our law which required judicial proceedings ● DSWD recommended for approval of the petition on the
for adoption. Limitation of registration of adoptions to those basis of its evaluation that the Millers were morally,
granted by Philippine courts is a misconception. For, if registration is emotionally and financially fit to be adoptive parents and
to be narrowed down to local adoptions, it is the function of the adoption would be the minor’s best interest and
Congress, not SC, to spell out such limitation. The Court cannot welfare.
carve out a prohibition where the law does not so state. Excessive RTC: Granted the petition for adoption
rigidity serves no purpose. And, ​by Articles 407 and 408 of our Civil
Code, the disputed document of adoption is registrable​. ● SolGen, in behalf of the Republic, filed an appeal to the CA
which certified the case to the SC
ISSUE
The record does not show that prejudice would ensure to State and
adoptee, or any other person for that matter, from the adoption WON aliens are qualified to adopt despite the prohibition under the
here involved. The validity thereof is not under attack. At any rate, Family Code
whatever may be the effect of adoption, ​the rights of the State and
adoptee and other persons interested are fully safeguarded by
Article 15 of our Civil Code which provides that: ​"Laws relating to HELD
family rights and duties, or to the status, condition and legal YES
capacity of persons are binding upon citizens of the Philippines
even though living abroad." ● An alien who filed a petition for adoption before the
effectivity of the Family Code, although denied the right to
adopt under Art. 184 of the Code, may continue with his
Private international law offers no obstacle to recognition of foreign petition under the law prevailing before the FC.
adoption. This rests on the principle that the status of adoption, ● The issue is not new. The Court has ruled than an alien is
created by the law of a State having jurisdiction to create it, will be qualified to adopt under the Child and Youth and Welfare
given the same effect in another state as is given by the latter state Code (CYWC). The petition was filed on July 29, 1988 prior
to the status of adoption when created by its own law. ​The status of to the effectivity of the FC. It was still governed by the
adoption, once created under the proper foreign law, will be CYWC which allowed aliens to adopt.
recognized in this country, except where public policy or the ● The enactment of the FC will not impair the right of an
interests of its inhabitants forbid its enforcement and demand the alien under the CYWC to adopt a Filipino child as this is a
substitution of the lex fori. ​Indeed, implicit in Article 15 of our Civil vested right at the time of the filing of the petition for
Code just quoted, is that the exercise of incidents to foreign adoption. The petition is governed by the law then in force
adoption "remains subject to local law." when it was filed.
WHEREFORE, we AFFIRM the appealed decision of the Regional Trial
It is high time for this Court to formulate a rule on the registration of Court.
foreign adoptions. We hold that ​an adoption created under the law
of a foreign country is entitled to registration in the corresponding Succession
civil register of the Philippines. It is to be understood, however,
that the effects of such adoption shall be governed by the laws of 21. Allison G. Gibbs v. Government of the Philippine Islands, GR
this country. No. L-35694, December 23, 1933

Facts:
CFI decision ​REVERSED and the Local Civil Registrar of Manila is
directed to register the deed of adoption.
Gibbs and his wife were American nationals, domiciled in
California. They acquired lands in the Philippines. The wife died in
20. Republic vs. Miller 306 SCRA 183 (1999) California. Gibbs was appointed administrator of the intestate
proceedings instituted in Manila.
REPUBLIC v. MILLER
G.R. No. 125932 | April 21, 1999 | J. Pardo Gibbs asked the court to adjudicate to him lands acquired in
the Philippines not under our laws on succession but because in
accordance with the law of California, the community property of
FACTS
spouses who are citizens of California, upon the death of the wife
● Spouses Claude and Jumrus Miller are 38 and 40 years of previous to that of the husband, belongs absolutely to the surviving
age, respectively, both American citizens. They were husband without administration. CFI granted such to Gibbs upon
childless and do not expect to have sibling out of their proof of California law.
union on account of a medical problem of the wife.
● Claude was a member of the US Air Force as airman first The register of deeds refused to transfer such properties on
class assigned at Clark Air Base and they maintain a the ground of non-payment of inheritance tax. Gibbs argued that
residence at Don Bonifacio Subdivision, Angeles. the conjugal right of a California wife in a community property is a
personal right and even if this was a case of succession, California
law would still apply.
“Nevertheless,legal and testamentary successions, in respect
Issue​: Was Eva Gibbs, at the time of her death, the owner of a to the order of succession as well as to the amount of the
successional rights and the intrinsic validity of their provisions, shall
descendible interest in the Ph?
be regulated by the national law ,of the person whose succession is
in question, whatever may be the nature of the property or the
Held: country in which it may be situated."(Art.10, second paragraph, OLD
CC)
Yes. The court held that it is principle firmly established that to
the law of the state in which the land is situated we must look for
The wife of the appellee was, by the law of the Philippine
the rules which govern its descent, alienation, and transfer, and for
the effect and construction of wills and other conveyances. This Islands, vested of a descendible interest, equal to that of her
fundamental principle is stated in the first paragraph of article 10 of husband, in the Philippine lands covered by certificates of title Nos.
our Civil Code(the old one) as follows: 20880, 28336 and 28331, from the date of their acquisition to the
date of her death. It results that the wife of the appellee was, by the
law of the Philippine Islands, vested of a descendible interest, equal
"Personal property is subject to the laws of the nation of the to that of her husband and the descendible interest of Eva Johnson
owner thereof; real property to the laws of the country in which it is Gibbs in the lands aforesaid was transmitted to her heirs by virtue of
situated.(Lex rei sitae) inheritance and this transmission plainly falls within the language of
section 1536 of Article XI of Chapter 40 of the Administrative Code
Under the provisions of the Civil Code and the jurisprudence which levies a tax on inheritances.
prevailing here, the wife, upon the acquisition of any conjugal
property, becomes immediately vested with an interest and title
It is unnecessary in this proceeding to determine the "order of
therein equal to that of her husband, subject to the power of
management and disposition which the law vests in the husband. succession" or the "extent of the successional rights" (article 10, Civil
Code, ​supra) ​which would be regulated by section 1386 of the Civil
Code of California which was in effect at the time of the death of
Mrs. Gibbs.
Under the law of California(to which Mr.Gibbs anchors his
claim of exemption to the payment of inheritance tax), upon the
death of the wife, the entire community property without
administration belongs to the surviving husband; that he is the
absolute owner of all the community property from the moment of
the death of his wife, not by virtue of succession or by virtue of her
death, but by virtue of the fact that when the death of the wife
precedes that of the husband he acquires the community property,
not as an heir or as the beneficiary of his deceased wife, but because
she never had more than an inchoate interest or expectancy which 22. Testate Estate of Amos G. Bellis v. Edward A. Bellis, ​supra at 4.​
is extinguished upon her death.
FACTS:

"The decisions under this section (1401 Civil Code of


● Amos Bellis was a citizen of the State of Texas, and of the
California) are uniform to the effect that the husband does not take
United States.
the community property upon the death of the wife by succession,
● By his first wife whom he divorced he had five legitimate
but that he holds it all from the moment of her death as though
children (Edward Bellis et al), by his second wife, who
acquired by himself.
survived him, he had three legitimate children.
● He, however, also had three illegitimate children in the
Philippines (Maria Cristina Bellis et al).
● Before he died, he made two wills, one disposing of his
The second paragraph of article 10, Civil Code, applies only Texas properties and the other disposing his Philippine
when a legal or testamentary succession has taken place in the properties.
Philippines in accordance with the law of the Philippine Islands; and ● In both wills, his illegitimate children were not given
the foreign law is consulted only in regard to the order of succession anything.
or the extent of the successional rights; in other words, the second ● The illegitimate children opposed the will on the ground
paragraph of Article 10 can be invoked only when the deceased was that they have been deprived of their legitimes to which
vested with a descendible interest in property within the jurisdiction they should be entitled, if Philippine law were to be
of the Philippine Islands. applied.

ISSUE:
The nature and extent of the title which vested in Mrs. Gibbs
at the time of the acquisition of the community lands here in
question must be determined in accordance with the ​lex rei sitæ. Whether the Philippine law be applied in the case in the
(Following this line of reasoning, lex rei sitae muna in accordance determination of the illegitimate children’s successional rights. ​-NO.
with par.1 of Art 10, then check the foreign law with regard doon sa Texas law will be applied.
order of succession or extent of successional rights in accordance
with par.2) -sana tama yung pagkakaintindi ko RULING:
● The Court ruled that provision in a foreigner’s will to the 9. June 12, 1916 - Lawyers of Ebba entered an appearance
and opposed the will being admitted to probate. CFI
effect that his properties shall be distributed in accordance
denied.
with Philippine law and not with his national law, is illegal
10. Ebba argues that ​Emil H. Johnson was a resident of the city
and void, for ​his national law (TEXAS LAW) cannot be
of Manila and not a resident of the State of Illinois at the
ignored in view of those matters that Article 10 — now
time the will in question was executed; The will is invalid
Article 16 — of the Civil Code states said national law
and inadequate to pass real and personal property in the
should govern.
State of Illinois; The order admitting the will to probate
● Where the testator was a citizen of Texas and domiciled in
was made without notice to the petitioner; and the order
Texas, the intrinsic validity of his will should be governed
in question was beyond the jurisdiction of the court.
by his national law. ​Since Texas law does not require
legitimes, then his will, which deprived his illegitimate Issue:
children of the legitimes, is valid. Whether or not there was deprivation of due process on the part
● The Supreme Court held that the ​illegitimate children are
of the petition
not entitled to the legitimes under the Texas law, which
is the national law of the deceased. ● Held: No. Due publication was made pursuant to this order
of the court through the three-week publication of the
23. In the Matter of the Estate of Emil H. Johnson, ​supra at 40​ notice in Manila Daily Bulletin. The Supreme Court also
asserted that in view of the statute concerned which reads
FACTS: as “A will made within the Philippine Islands by a citizen or
1. Feb 4, 1916 - Johnson, a swedish native and a naturalized subject of another state or country, which is executed in
accordance with the law of the state or country of which
citizen of the US, died in Manila leaving a will dated Sept
he is a citizen or subject, and which might be proved and
9, 1915 where he disposed of an estate with the value of
allowed by the law of his own state or country, may be
P231,800. The will was holographic (written in his own
proved, allowed, and recorded in the Philippine Islands,
handwriting, and is signed by himself and two witnesses
and shall have the same effect as if executed according to
only, instead of 3 as required by Sec 618 of the Code of
the laws of these Islands” the “state”, being not
Civil Procedure. (will was not executed in conformity with
capitalized, does not mean that United States is excluded
the provisions of law generally applicable to wills and
from the phrase (because during this time, Philippines was
could not be have been proved under sec 618)
still a territory of the US).
2. Feb 9, 1916 - A Petition was filed before the CFI of Manila
for the probate of his will on the ground that Johson was
a citizen of the state of Illinois, USA; the will was duly WON Section 636 of the Code of Civil Procedure is not applicable to
executed in accordance with the law of Illinois; and thus wills of aliens residing in RP?
could be probated under Sec 636 of the Code of Civil ● Section 636: authorizes probate by our courts of a will
Procedure. made within the Philippine Islands by a citizen or subject
3. Sec 636 provides: ​A will made within the Philippine Islands of another ​state or country, when such will is executed in
by a citizen or subject of another state or country, which is accordance with the law of the ​state or country of which
executed in accordance with the law of the state or the testator is a citizen of subject, and which might be
country of which he is a citizen or subject, and which proved under the law of such ​state​ or country.
might be proved and allowed by the law of his own state ● IT IS APPLICABLE: the "state" would include US, and the
or country, may be proved, allowed, and recorded in the operation of law is not limited to wills of aliens.
Philippine Islands, and shall have the same effect as if ● If Johnson was at the time of his death a citizen of US and
executed according to the laws of these Islands. of the state of Illinois, his will was provable under this
4. Court found the document to be legal and was admitted to section in the courts of the Philippines, provided the
probate. Victor Johnson was the sole administrator. instrument was so executed as to be admissible to probate
5. the will in question the testator gives to his brother Victor under laws of the State of Illinois
one hundred shares of the corporate stock in the
Johnson-Pickett Rope Company; to his father and mother WON Decedent a national of Illinois (to warrant the application of
in Sweden, the sum of P20,000; to his daughter Ebba Illinois law)
Ingeborg, the sum of P5,000; to his wife, Alejandra Ibañez, ● YES. Proof adduced before TC showed he was indeed a
the sum of P75 per month, if she remains single; to national of Illinois. Petition merely contests the residence
Simeona Ibañez, spinster, P65 per month, if she remains of the decedent to be in the Philippines, but not the
single. The rest of the property is left to the testator's five nationality
children — Mercedes, Encarnacion, Victor, Eleonor and ● Why contested residence: US naturalization laws require
Alberto residence of at least 5 years in US and 1 year w/n the State
6. Emil Johnson was born in sweden, migrated to the US and or territory where the court granting the naturalization
lived in Chicago. In 1898, he married Rosalie, and then papers is held to grant the certificate of naturalization
went to the PH as a soldier for the US army. Rosalie and ● -still, no other proof to rebut the presumption that he was
Emil had a daughter named Ebba indeed naturalized as a US citizen (particularly of Illinois)
7. After being discharged as a soldier, he contnued to live in
the PH. Rosalie got a divorce. Emil went back to Chicago WON will executed in conformity with the State of Illinois
and got a certificate of naturalization. He then visited ● NOT REALLY SURE, BUT THE PETITIONER CANNOT DO
Sweden, and live the rest of his life in the PH. ANYTHING ABOUT IT.
8. In the PH, he entered into marital relations with Alejandra ● Courts cannot take JN of Foreign laws: TC merely relied on
which resulted to 3 children. The other 2 were borne to the ​presentation of Section 1874 of the Revised Statutes
the deceased Simeona. of Illinois as exhibited in a volume of an annotation and
assumed that he could take JN of the laws of Illinois. But it
was mistaken
● Proper rule is to require proof of the statutes of the States ISSUE:
of the American Union whenever their provisions are
determinative of the issues in any action litigated in the WON the testamentary dispositions, especially those for the children
Philippine courts. which are short of the legitime given them by the Civil Code of the
● Still, (1) petition does not state any fact from which it
Philippines, are valid?
would appear that the law of Illinois is different from what
the court found and (2)petition did not raise any RULING:
assignment of error to question the supposed taking of JN
of the court ● The old Civil Code, which is applicable to this case because
the testator died in 1944, expressly provides that
successional rights to personal property are to be earned
24. Juan Miciano v. Andre Brimo, G.R. No. L-22595/50 Phil 867, 1
by the national law of the person whose succession is in
November 1927;
question. Says the law on this point: Nevertheless, legal
and testamentary successions, in respect to the order of
25. Francisco Carmelo Varela v. Miguel Varela Calderon, G.R. No. succession as well as to the extent of the successional
L-36342 / 57 Phil 280, 8 October 1932; rights and the intrinsic validity of their provisions, shall be
regulated by the national law of the person whose
26. Philippine Trust Co. v. Magdalena C. Bohanan​,supra.; ​ succession is in question, whatever may bethe nature of
the property and the country in which it is found.
DOCTRINE: The validity of testamentary dispositions are to be
governed by the national law of the testator, provided that the law ● The testator was a citizen of the State of Nevada because
must be proved incourts. he had selected this as his domicile and his permanent
CONFLICT LAWS: Old CC Art. 10(2), now NCC Art. 16(2) residence. It does not appear that at time of the hearing of
“Nevertheless, legal and testamentary successions, in respect to the the project of partition, the above-quoted provision was
order of succession as well as to the extent of the successional rights introduced in evidence.
to personal property are to be earned by the national law of the ● During the hearing of the motion of Magdalena C.
person whose succession is in question.”Nevada Compiled Laws of Bohanan for withdrawal of P20,000 asher share, the
1925, Sec. 9905 “Every person over the age of 18 years, of sound
mind, may, by last will,dispose of all his or her estate, real and foreign law, especially Section 9905,Compiled Nevada
personal, the same being chargeable with the payment of the Laws was introduced in evidence by appellant's (herein)
testator’s debts.” counsel. Again said laws presented by the counsel for the
FACTS: executor and admitted by the Court during the hearing of
● Testator Bohanan was born in Nebraska and was a US the case.In addition, the other appellants, children of the
citizen. He has some properties in California. Despite his testator,do not dispute the above-quoted provision of the
long residence in the Philippines,his stay was found by the laws of the State of Nevada. Under all the above
CFI to be merely temporary, and he remained to be a US circumstances,we are constrained to hold that the
citizen. pertinent law of Nevada, especially Section 9905 of the
● The CFI declared his will as fully in accordance with the
laws of Nevada and admitted it to probate. The Philippine Compiled Nevada Laws of 1925, can be taken judicial
Trust Co. was named executor of the will. notice of by us, without t proof of such law having been
● Accordingly, the Philippine Trust Company, named as the offered at the hearing of the project of partition. As in
executor of the will, was appointed to such executor and accordance with Article 10 of the old Civil Code, the
upon the filing of a bond in the sum of P10,000.00.The validity of testamentary dispositions are to be governed by
executor filed a project of partition making, in accordance
with the provisions of the will, the following adjudications: the national law of the testator, and as it has been decided
(1) one-half of the residuary estate, to the Farmers and and it is not disputed that the national law of the testator
Merchants National Bank of Los Angeles, intrust only for is that of the State of Nevada, already indicated above,
the benefit of testator's grandson which consists of several which allows a testator to dispose of all his property
mining companies; (2) the other half of the residuary
according to his will, as in the case at bar, the order of the
estate to the testator's brother and his sister(3) legacies of
P6,000 each to his (testator) son and his daughter to be court approving the project of partition made in
paid in three yearly installments; (4)legacies to *others accordance with the testamentary provisions, must be, as
irrelevant to the case*Out of the total estate of it is hereby affirmed, with costs against appellants
P211,639.33 in cash, the testator gave his grandson
P90,819.67 and one-half of all shares of stock of several
mining companies and to his brother and sister the same
amount. To his children he gave a legacy of only P6,000
each, or a total of P12,000. 27. Adolfo Aznar v. Helen Christensen Garcia​, supra.;
● Respondent Magdalena Bohanan, his ex-wife, questions
the validity of the partition, claiming that she and her
children were deprived of their legitimes. FACTS:
● (It must be noted that Magdalena and decedent C.O. ● Edward Christensen, a US citizen and domiciled in the
Bohanan were married in 1909 but he divorced her in Philippines, left a will wherein he instituted an
1922. She remarried in 1925 and this marriage was acknowledged daughter, Maria Lucy Christensen, as his
subsisting at the time of the death of decedent only child.
● He also bequeathed P3,600.00 to Maria Helen Christensen, benefit and therefore not valid, and that it was not executed
who is a baptized Christensen, but is not related or has not and attested to as required by law.
been adopted at any time by the testator. · After hearing, the trial court found the will to have been
● Helen opposed the partition as it deprives her of her
executed conjointly by the deceased husband and wife for their
legitime as an acknowledged natural child and that the
reciprocal benefit, and that a will of that kind is neither
distribution should be governed by the laws of the
contemplated by Act No. 190, known as the Code of Civil
Philippines.
Procedure nor permitted by article 669 of the Civil Code which
○ Helen claims that under Art. 16, par. 2 of the
provides:
Civil Code, California law should be applied; that
under California law, the matter is referred back "Two or more persons cannot make a will
to the law of the domicile. conjointly or in the same instrument, either for
● Maria averred that the national law of the deceased must their reciprocal benefit or for the benefit of a
apply, illegitimate children not being entitled to anything third person."
under California law.

ISSUE: ​Whether or not the lower court erred in finding that a joint
ISSUE: and reciprocal will particularly between husband and wife is void
Whether the national law of the deceased should be applied in under the present law. – NO, it really is invalid.
determining the successional rights of his heirs
HELD: ​According to Ramona, sections 614 and 618 of Act No. 190
RULING: appear to be a complete enactment on the subject of execution of
wills and may thus be regarded as the expression of the whole law
The Court initially ruled that the national law applies but it was thereon, and that it must be deemed to have impliedly repealed the
referred back to domiciliary law. provision of the Civil Code. Joint and reciprocal wills are neither
regarded as invalid nor on the contrary they are allowed, then article
669 of the Civil Code prohibiting the execution of joint wills whether
Article 16 of the Civil Code provides that the intrinsic validity of reciprocal or for the benefit of a third party should be considered as
testamentary dispositions is governed by the national law of the having been repealed and superseded by the new law. We cannot
decedent, which in this case, California law. There are two agree to the contention of Ramona. In the study made on this
conflicting California laws regarding succession: one that does not subject, We have found a number of cases decided by this court
provide for legitimes and one that provides that the law of the wherein several articles of the Civil Code regarding wills have not
domicile applies (Art. 946 of the California Civil Code). Art. 946 as only been referred to but have also been applied side by side with
the applicable law and in accordance therewith and following the the provisions of the Code of Civil Procedure.
doctrine of renvoi, the question of the validity of the testamentary
provision in question should be referred back to the law of the
decedent’s domicile which is the Philippines. Thus, Helen is entitled Another contention of Ramona is that in the case of ​Ham vs.
to her legitime. Saavedra​, a will was executed jointly by husband and wife in a same
instrument, and it was admitted to probate. The decision was also
affirmed by the Supreme Court, thereby proving that this tribunal
has disregarded the prohibition regarding the execution of wills
28. Ramon N. Bilbao v. Dalmacio Bilbao, G.R. No. L-2200/87 Phil conjointly under article 669 of the Civil Code, meaning that said
article has already been repealed. This is untenable. It is true that
144, 2 August 1950;
the will already described was allowed probate by the trial court,
28 – Ramon Bilbao vs. Dalmacio, Cleofas, Eusebia, and Catalina but there was no appeal from the order approving the will on the
Bilbao, Filemon Abringe, and Francisco Academia ground of its invalidity, but only on the manner the properties
involved were to be distributed or otherwise disposed of. The
G.R. No. L-2200 | 1950-08-02 | Montemayor, J. Supreme Court never touched this point of invalidity nor the
applicability of article 669 of the Civil Code.

FACTS:
The provision of article 669 of the Civil Code prohibiting the
· This is an appeal from the CFI Negros Oriental denying the execution of a will by two or more persons conjointly or in the same
petition for admission to probate of the last will and testament instrument either for their reciprocal benefit or for the benefit of a
of Victor S. Bilbao who died in 1943, which was filed by his third person, is not unwise and is not against public policy. ​The
widow and co-testator Ramona Navarro. reason for this provision, especially as regards husbands and wife
· The will in question was executed in 1931, on a single page by is that when a will is made jointly or in the same instrument, the
the deceased Victor Bilbao jointly with his wife Ramona M. spouse who is more aggressive, stronger in will or character and
Navarro. The two testators in their testament directed that "all dominant is liable to dictate the terms of the will for his or her own
of our respective private properties both real and personal, and benefit or for that of third persons whom he or she desires to
all of our conjugal properties, and any other property belonging favor. And, where the will is not only joint but reciprocal, either
to either or both of us, be given and transmitted to anyone or one of the spouses who may happen to be unscrupulous, wicked,
either of us, who may survive the other, or who may remain the faithless or desperate, knowing as he or she does the terms of the
surviving spouse of the other." will whereby the whole property of the spouses both conjugal and
paraphernal goes to the survivor, may be tempted to kill or dispose
· The petition for probate was opposed by Filemon Abringe, a of the other.
near relative of Victor Bilbao on the ground that the alleged will
was executed by the husband and wife for their reciprocal
The CA decision is affirmed.

30. Polly Cayetano vs. Hon. Tomas T. Leonidas, G.R. No.


29. Severina A. vda. De Enriquez v. Miguel Abadia, G.R. No. L-54919/129 SCRA 522, 30 May 1984;
L-7188/95 Phil 627 9 August 1954;
Facts:
FACTS:
● Father Sancho Abadia executed a document purpoting to 1. Adoracion C. Campos, in her lifetime, was a citizen of the United
be his last will and testament. When Father Abadia died, States of America and a permanent resident of Philadelphia. She
one of the legatees, Andres Enriquez filed a petition for executed a Last Will and Testament in the county of
probate of the alleged will. This was opposed by the Philadelphia,Pennsylvania, U.S.A., according to the laws thereat, and
nephews and nieces of the deceased who will be able to that while in temporary sojourn in the Philippines.
inherit if there was no will.
● The trial court ruled that the document is a holographic 2​. Adoracion C. Campos died in the City of Manila, leaving property
will. It was in the handwriting of the testator and that both in the Philippines and in the United States of America.
although at the time it was executed and at the time of the
testator's death, holographic wills were not permitted by 3. The Last Will and Testament of the late Adoracion C. Campos was
law still, because at the time of the hearing and when the admitted and granted probate by the Orphan's Court Division of the
case was to be decided the new Civil Code was already in Court of Common Pleas, the probate court of the Commonwealth of
force, which Code permitted the execution of holographic Pennsylvania, County of Philadelphia, U.S.A.
wills, under a liberal view, and to carry out the intention of
the testator which according to the trial court is the 4. The letters of administration were issued in favor of Clement J.
controlling factor and may override any defect in form. McLaughlin all in accordance with the laws of the said foreign
country on procedure and allowance of wills. Nenita C. Paguia,
daughter or the testator, was appointed Administratrix of the estate
Issue: ​What law will govern? (old civil code v new civil code) of said decedent.

5. This was opposed by Adoracion’s father, Hermogenes Campos​,


Held: Old Civil Code who earlier filed an Affidavit of Self-adjudication whereby he
Article 795 of the new Civil Code expressly provides: "The validity of adjudicated unto himself the ownership of the entire estate of the
deceased Adoracion Campos, not being aware that Adoracion had
a will as to its form depends upon the observance of the law in force
left a will. He later died and was substituted by Polly Cayetano as
at the time it is made." The above provision is but an expression or
petitioner in the instant case.
statement of the weight of authority to the affect that the validity of
a will is to be judged not by the law enforce at the time of the
6​. A motion to dismiss the petition on the ground that the rights of
testator's death or at the time the supposed will is presented in
the petitioner Hermogenes Campos merged upon his death with the
court for probate or when the petition is decided by the court but at
rights of the respondent and her sisters, only remaining children and
the time the instrument was executed. One reason in support of the
forced heirs was denied on September 12, 1983.
rule is that although the will operates upon and after the death of
the testator, the wishes of the testator about the disposition of his
7. Cayetano alleged that the trial court erred in ruling that the right
estate among his heirs and among the legatees is given solemn
of a forced heir to his legitime can be divested by a decree
expression at the time the will is executed, and in reality, the legacy
admitting a will to probate in which no provision is made for the
or bequest then becomes a completed act.
forced heir​ incomplete disregard of Law of Succession.
There is the view that the intention of the testator should be the
ruling and controlling factor and that all adequate remedies and
ISSUE: Whether or not a forced heir is entitled to his legitime in
interpretations should be resorted to in order to carry out said
case the testator was a citizen of another country.
intention, and that when statutes passed after the execution of the
will and after the death of the testator lessen the formalities
RULING: NO.
required by law for the execution of wills, said subsequent statutes
should be applied so as to validate wills defectively executed
The private respondents have sufficiently established that Adoracion
according to the law in force at the time of execution. However, we
was, at the time of her death, an American citizen and a permanent
should not forget that from the day of the death of the testator, if
resident of Philadelphia, Pennsylvania, U.S.A. Therefore, under
he leaves a will, the title of the legatees and devisees under it
Article 16 par. (2) and 1039 of the Civil Code which respectively
becomes a vested right, protected under the due process clause of
provide:
the constitution against a subsequent change in the statute adding
Art. 16 par. (2).
new legal requirements of execution of wills which would invalidate
xxx xxx xxx
such a will. By parity of reasoning, when one executes a
will which is invalid for failure to observe and follow the legal
requirements at the time of its execution then upon his death he However, intestate and testamentary successions, both with
should be regarded and declared as having died intestate, and his respect to the order of succession and to the amount of
heirs will then inherit by intestate succession, and no subsequent successional rights and to the intrinsic validity of testamentary
law with more liberal requirements or which dispenses with such provisions, shall be regulated by the national law of the person
requirements as to execution should be allowed to validate a whose succession is under consideration, whatever may be the
defective will and thereby divest the heirs of their vested rights in nature of the property and regardless of the country wherein
the estate by intestate succession. The general rule is that the said property may be found.
Legislature can not validate void wills
Art. 1039. execution.

Capacity to succeed is governed by the law of the nation of the


decedent. RULING

YES. Section 635 of the Code of Civil Procedure stating that “a will
The law which governs Adoracion Campo's will is the law of
made out of the Philippine Islands… may be proved, allowed, and
Pennsylvania, U.S.A., which is the national law of the decedent.
recorded in the Philippine Islands, and shall have the same effect
Although the parties admit that the Pennsylvania law does not
as if executed according to the laws of these Islands” is still in force
provide for legitimes and that all the estate may be given away by
and has not been abrogated by Rule 78 of the Rules of Court. Here,
the testatrix to a complete stranger, the petitioner argues that such
the will of William Giberson need not be probated first in the State
law should not apply because it would be contrary to the sound
of Illinois, USA before it may be probated here in the Philippines.
and established public policy and would run counter to the specific
The Court opined that Section 635 of the Code of Civil Procedure is
provisions of Philippine Law.
substantive in nature and therefore could not have been repealed
It is a settled rule that as regards the intrinsic validity of the by the Rules of Court which are only procedural in nature.
provisions of the will, as provided for by Article 16(2) and 1039 of
the Civil Code, the national law of the decedent must apply. This
was squarely applied in the case of Bellis v. Bellis (20 SCRA 358) 32. Miciano vs. Brimo (50 Phil 867 (1927)
wherein we ruled: FACTS: ​The partition of the estate left by the deceased Joseph G.
Brimo is in question in this case.
It is therefore evident that whatever public policy or The judicial administrator of this estate filed a scheme of partition.
good customs may be involved in our system of
Andre Brimo, one of the brothers of the deceased, opposed it. The
legitimes, Congress has not intended to extend the
court, however, approved it.
same to the succession of foreign nationals. For it has
specifically chosen to leave, inter alia, the amount of The errors which the oppositor-appellant assigns are:
successional rights, to the decedent's national law. (1) The approval of said scheme of partition; (2) denial of his
Specific provisions must prevail over general ones.
participation in the inheritance; (3) the denial of the motion for
reconsideration of the order approving the partition; (4) the
xxx xxx xxx approval of the purchase made by the Pietro Lana of the deceased's
The parties admit that the decedent, Amos G. Bellis, was business and the deed of transfer of said business; and (5) the
declaration that the Turkish laws are impertinent to this cause, and
a citizen of the State of Texas, U.S.A., and under the law
the failure not to postpone the approval of the scheme of partition
of Texas, there are no forced heirs or legitimes.
and the delivery of the deceased's business to Pietro Lanza until the
Accordingly, since the intrinsic validity of the provision
receipt of the depositions requested in reference to the Turkish
of the will and the amount of successional rights are to
laws.
be determined under Texas law, the Philippine Law on
legitimes cannot be applied to the testacy of Amos G. The appellant's opposition is based on the fact that the partition in
Bellis. question puts into effect the provisions of Joseph G. Brimo's will
which are not in accordance with the laws of his Turkish nationality,
for which reason they are void as being in violation or article 10 of
Therefore, the settlement of the estate of Adoracion Campos was
the Civil Code
correctly filed with the Court of First Instance of Manila where she
had an estate since it was alleged and proven that Adoracion at the ISSUE: ​Whether the Turkish Law or Philippine Law be the basis on
time of her death was a citizen and permanent resident of the distribution of Joseph Brimo's estates. Will Andre Brimo forfeit
Pennsylvania, United States of America and not a "usual resident of his inheritance?
Cavite" as alleged by the petitioner. HELD: Respondent did not prove that said testamentary dispositions
are not in accordance with the Turkish laws, inasmuch as he did not
present any evidence showing what the Turkish laws are on the
31. Dalton vs. Giberson 91 Phil 524 (1952) matter, and in the absence of evidence on such laws, they are
presumed to be the same as those of the Philippines.
FACTS
It has not been proved in these proceedings what the Turkish laws
are. He, himself, acknowledges it when he desires to be given an
Lela G. Dalton presented on February 10, 1949 an application with
opportunity to present evidence on this point; so much so that he
the Court of First Instance of Cebu for the probate of the
assigns as an error of the court in not having deferred the approval
holographic will of William R. Giberson, a citizen of the State of
of the scheme of partition until the receipt of certain testimony
Illinois, United States, dated April 29, 1920 in San Francisco,
requested regarding the Turkish laws on the matter.
California. Spring Giberson, legitimate son of William R. Giberson,
presented an opposition alleging that the will is apocrypha (with The approval of the scheme of partition in this respect was not
questionable authenticity), it does not represent the true will of erroneous.
the late Giberson, and has not been granted according to the law. The institution of legatees in this will is conditional, and the
condition is that the instituted legatees must respect the testator's
will to distribute his property, not in accordance with the laws of his
nationality, but in accordance with the laws of the Philippines.
ISSUE
If this condition as it is expressed were legal and valid, any legatee
Whether the wills executed outside the Philippines may be who fails to comply with it, as the herein oppositor who, by his
probated without being first probated in the country of its
attitude in these proceedings has not respected the will of the ○ Magdalena was not given in the will any share in
testator, as expressed, is prevented from receiving his legacy.
the estate left
The fact is, however, that the said condition is void, being contrary ○ Children received legacies in the amount of
to law, for article 792 of the civil Code provides the following: P6,000 each only
Impossible conditions and those contrary to law or · It is further argued that it was error for the trial court to have
good morals shall be considered as not imposed and recognized the Reno divorce secured by the testator from his
shall not prejudice the heir or legatee in any manner Filipino wife Magdalena C. Bohanan, and that said divorce
whatsoever, even should the testator otherwise should be declared a nullity in this jurisdiction
provide. · Lower court: refused to recognize the claim of the widow;
And said condition is contrary to law because it expressly ignores the testamentary dispositions in the will
testator's national law when, according to article 10 of the civil Code ○ That the laws of Nevada US, of which the
above quoted, such national law of the testator is the one to govern
deceased was a citizen, allow him to dispose of
his testamentary dispositions.
all of his properties without requiring him to
Said condition then, in the light of the legal provisions above cited, is leave any portion of his estate to his wife
considered unwritten, and the institution of legatees in said will is ○ Section 9905 of Nevada Compiled Laws of 1925
unconditional and consequently valid and effective even as to the provides: Every person over the age of eighteen
herein oppositor. years, of sound mind, may, by last will, dispose
It results from all this that the second clause of the will regarding the of all his or her estate, real and personal, the
same being chargeable with the payment of the
law which shall govern it, and to the condition imposed upon the
testator's debts.
legatees, is null and void, being contrary to law.
○ That there is no right to share in the inheritance
All of the remaining clauses of said will with all their dispositions and in favor of a divorced wife exists in the State of
requests are perfectly valid and effective it not appearing that said Nevada and since the court below had already
clauses are contrary to the testator's national law. found that there was no conjugal property
Therefore, the orders appealed from are modified and it is directed between the testator and Magdalena
that the distribution of this estate be made in such a manner as to · Note however that the most important issue is the claim of the
include the herein appellant Andre Brimo as one of the legatees, and testator's children
the scheme of partition submitted by the judicial administrator is
approved in all other respects, without any pronouncement as to
costs. ISSUES:
(1) ​WON the testamentary dispositions, especially hose for the
33. Bohanan vs. Bohanan 106 Phil 1997 (1960) children which are short of the legitime given them by the Civil Code
of the Philippines, are valid - YES
TESTATE ESTATE OF C. O. BOHANAN, deceased. PHILIPPINE TRUST
CO., ​executor-appellee,
HELD:
vs.
(1) YES; Affirmed.
MAGDALENA C. BOHANAN, EDWARD C. BOHANAN, and MARY
LYDIA BOHANAN,​ oppositors-appellants. · The old Civil Code, which is applicable to this case because the
G.R. No. L-12105 | January 30, 1960 | LABRADOR, J. testator died in 1944, expressly provides that ​successional
rights to personal property are to be earned by the national law
Topic: Succession of the person whose succession is in question​. Says the law on
this point:
“Nevertheless, legal and testamentary successions, in respect to the
DOCTRINE: ​Successional rights to personal property are to be
order of succession as well as to the extent of the successional rights
earned by the national law of the person whose succession is in
and the intrinsic validity of their provisions, shall be regulated by the
question
national law of the person whose succession is in question,
whatever may be the nature of the property and the country in
FACTS: which it is found. (par. 2, Art. 10, old Civil Code, which is the same as
par. 2 Art. 16, new Civil Code.)”
● 1944: C.O. Bohonan executed his last will and testament
· In the proceedings for the probate of the will, it was found out
● 1950: CFI Manila, in an order, admitted to probate said
and it was decided that the testator was a citizen of the State of
will. In the said order, the ff facts were established:
Nevada because he had selected this as his domicile and his
○ C.O. Bohanan was born in Nebraska therefore he
permanent residence. Moreover, It is not disputed that the
is a citizen of Nebraska and even though he lived
laws of Nevada allow a testator to dispose of all his properties
in the Philippines for a long period of time. he
by will.
still remained a citizen of the united states. C.O.
Bohanan eventually chose Nevada to spend the o The law of Nevada, being a foreign law can
rest of his days so therefore when he died. He only be proved in our courts in the form and
remained a citizen of the US. manner provided for by our Rules, which
· Meanwhile, (former) wife Magadalena C. Bohanan and her two are as follows:
children question the validity of the testamentary provisions SEC. 41. Proof of public or official record. — An official record or an
claiming that they have been deprived of their legitime in entry therein, when admissible for any purpose, may be evidenced
accordance with the law of the forum by an official publication thereof or by a copy tested by the officer
having the legal custody of he record, or by his deputy, and
accompanied, if the record is not kept in the Philippines, with a
certificate that such officer has the custody. . . . (Rule 123).
· The foreign law (Nevada Laws) was introduced in evidence by
appellant's counsel and was admitted by the Court as Exhibit
"B" during the hearing of the case. In addition, the other
appellants, children of the testator, do not dispute the
above-quoted provision of the laws of the State of Nevada.
Under all the above circumstances, we are constrained to hold
that the pertinent law of Nevada, especially Section 9905 of the
Compiled Nevada Laws of 1925, can be taken judicial notice of
by us, without proof of such law having been offered at the
hearing of the project of partition.
As in accordance with ​Article 10 of the old Civil Code, the validity of
testamentary dispositions are to be governed by the national law of
the testator​, and as it has been decided and it is not disputed that
the ​national law of the testator is that of the State of Nevada,
already indicated above, which allows a testator to dispose of all his
property according to his will​, as in the case at bar, the order of the
court approving the project of partition made in accordance with the
testamentary provisions, must be, as it is hereby affirmed, with costs
against appellants.

S-ar putea să vă placă și